West Coast EMT Chapter 26 -- Soft-Tissue Injuries, West Coast EMT Chapter 24 -- Trauma Overview, West Coast EMT Chapter 31 -- Orthopedic Injuries, West Coast EMT Chapter 30 -- Abdominal and Genitourinary Injuries, West Coast EMT Chapter 29 -- Chest I...

Pataasin ang iyong marka sa homework at exams ngayon gamit ang Quizwiz!

When the speed of a motor vehicle doubles, the amount of kinetic energy: A. quadruples. B. triples. C. doubles. D. is not affected.

A

Which of the following joints allows no motion? A. skull sutures B. sacroiliac joint C. shoulder joint D. sternoclavicular joint

A

Which of the following musculoskeletal injuries has the GREATEST risk for shock due to blood loss? A. pelvic fracture B. posterior hip dislocation C. unilateral femur fracture D. proximal humerus fracture

A

leave his helmet and shoulder pads in place

A high school football player was injured during a tackle and complains of neck and upper back pain. He is conscious and alert and is breathing without difficulty. The EMT should

pulmonary contusion

A man called EMS 12 hours after injuring his chest. Your assessment reveals a flail segment to the right side of the chest. The patient is experiencing respiratory distress and his oxygen saturation is 78%. His breath sounds are equal bilaterally and his jugular veins are normal. You should suspect:

cover the exposed bowel with a moist sterile dressing and then secure it in place with a dry bandage

A man has a large laceration across his lower abdominal wall and a loop of bowel is protruding from the wound. He is conscious and alert and there is minimal bleeding from the wound. You should:

axial loading

A man jumped from the roof of his house and landed on his feet. He complains of pain to his heels, knees, and lower back. This mechanism of injury is an example of:

retrograde amnesia

A patient who cannot remember the events that preceded his or her head injury is experiencing:

repeats BP of 92/68 mmHg

A patient with a chest injury has a blood pressure of 100/60 mm Hg and a pulse rate of 120 beats/min. Which of the following additional findings should make you suspect a pericardial tamponade?

3

A patient with a head injury presents with abnormal flexion of his extremities. What numeric value should you assign to him for motor response?

A young boy was riding his bicycle down the street when he hit a parked car. What was the first collision? A. The bike hitting the car. B. The bike rider hitting his bike or the car. C. The bike rider's internal organs against the solid structures of the body. D. The bike rider striking the pavement.

A

A young boy was riding his bicycle down the street when he hit a parked car. What will rate your index of suspicion for this collision? A. The mechanism of injury B. The type of bike C. How loudly he's crying D. A quick visual assessment

A

Significant clues to the possibility of severe injuries in motor vehicle collisions include: A. death of a passenger B. a blown out tires C. broken glass D. a deployed air bag

A

The MOST common and significant complication associated with fractures or dislocations of the knee is: A. neurovascular compromise. B. internal bleeding and shock. C. ligament and cartilage damage. D. total depletion of synovial fluid.

A

The MOST commonly fractured bone(s) in the body is the: A. clavicle. B. scapula. C. radius and ulna. D. midshaft femur.

A

The act of pulling on a body structure in the direction of its normal alignment is called: A. traction. B. reduction. C. stabilization. D. immobilization.

A

The bones in the skeleton produce _______ in the bone marrow. A. blood cells B. minerals C. electrolytes D. hormones

A

The energy of a moving object is called: A. kinetic energy. B. converted energy. C. potential energy. D. latent energy.

A

The musculoskeletal system refers to the: A. bones and voluntary muscles of the body. B. nervous system's control over the muscles. C. connective tissue that supports the skeleton. D. involuntary muscles of the nervous system.

A

The pectoral girdle consists of the: A. scapulae and clavicles. B. clavicles and rib cage. C. sternum and scapulae. D. acromion and clavicles.

A

The primary purpose for splinting a musculoskeletal injury is to: A. prevent further injury. B. maximize distal circulation. C. make the patient comfortable. D. facilitate ambulance transport.

A

In general, musculoskeletal injuries should be splinted before moving the patient unless: A. the patient is in severe pain. B. the patient is clinically unstable. C. deformity and swelling are present. D. transport time is less than 15 minutes.

B

The PASG is absolutely contraindicated in patients who have: A. pelvic fractures. B. pulmonary edema. C. bilateral femur fractures. D. any trauma below the pelvis.

B

The _______ is one of the most commonly fractured bones in the body. A. scapula B. clavicle C. humerus D. radius

B

The amount of kinetic energy that is converted to do work on the body dictates the _____ of the injury. A. location B. severity C. cause D. speed

B

The cervical spine is MOST protected from whiplash-type injuries when the: A. air bag correctly deploys upon impact. B. headrest is appropriately positioned. C. rear end of the vehicle is initially struck. D. patient tenses up at the time of impact.

B

What joint is frequently separated during football and hockey when a player falls and lands on the point of the shoulder? A. Glenohumeral joint B. Acromioclavicular joint C. Sternoclavicular joint D. None of the above

B

Which of the following injuries would be appropriate to treat with a traction splint? A. Injuries of the pelvis B. An isolated femur fracture C. Partial amputation or avulsions with bone separation D. Lower leg or ankle injury

B

Which of the following statements about compartment syndrome is FALSE? A. It occurs 6 to 12 hours after an injury. B. It most commonly occurs with a fractured femur. C. It is usually a result of excessive bleeding, a severely crushed extremity, or the rapid return of blood to an ischemic limb. D. It is characterized by pain that is out of proportion to the injury.

B

With regard to the three collisions that occur during a motor vehicle crash, which of the following statements regarding the first collision is correct? A. It provides the least amount of information about the mechanism of injury. B. It is the most dramatic part of the collision and may make extrication difficult. C. It occurs when the unrestrained occupant collides with the interior of the vehicle. D. It has a direct effect on patient care because of the obvious vehicular damage.

B

A patient with a head injury presents with abnormal flexion of his extremities. What numeric value should you assign to him for motor response? Select one: A. 5 B. 3 C. 2 D. 4

B. 3

Injuries to the external male genitalia _______. Select one: A. frequently lead to hypovolemic shock B. are rarely life-threatening C. usually result in permanent damage D. are often life-threatening

B. are rarely life-threatening

The Glasgow Coma Scale (GCS) is used to assess: Select one: A. verbal response, eye opening, and mental status. B. eye opening, verbal response, and motor response. C. sensory response, pupil reaction, and heart rate. D. mental status, eye opening, and respiratory rate.

B. eye opening, verbal response, and motor response.

Hyperextension injuries of the spine are MOST commonly the result of: Select one: A. diving. B. hangings. C. falls. D. compression.

B. hangings.

A young boy was riding his bicycle down the street when he hit a parked car. How many collisions took place? A. 1 B. 2 C. 3 D. 4

C

The most common life-threatening event in a rollover is ____ or partial ejection of the passenger from the vehicle. A. vehicle intrusion B. centrifugal force C. ejection D. spinal cord injury

C

The phenomenon of pressure waves emanating from the bullet, causing damage remote from its path, is known as: A. conversion. B. congruent. C. cavitation. D. capitation.

C

Signs and symptoms of sprains include all of the following EXCEPT: A. point tenderness. B. pain preventing the patient from moving or using the limb normally. C. marked deformity. D. instability of the joint indicated by increased motion.

C

When a motor vehicle strikes a tree while traveling at 40 mph, the unrestrained occupant: A. is thrust under the steering column onto the floorboard. B. will decelerate at the same rate as the motor vehicle. C. remains in motion until acted upon by an external force. D. will most likely be thrown over the steering column.

C

When a light is shone into the pupil: Select one: A. it should become smaller in size. B. both pupils should dilate together. C. it should become larger in size. D. the opposite pupil should dilate.

A. it should become smaller in size.

The superficial temporal artery can be palpated: Select one: A. just anterior to the tragus. B. slightly above the ear. C. at the angle of the jaw. D. over the mastoid process.

A. just anterior to the tragus.

Immediately life-threatening chest injuries must be found and managed during the _______. Select one: A. primary assessment B. scene size-up C. patient history D. secondary assessment

A. primary assessment

When activated, the sympathetic nervous system produces all of the following effects, EXCEPT: Select one: A. pupillary constriction. B. increase in heart rate. C. dilation of the bronchiole smooth muscle. D. shunting of blood to vital organs.

A. pupillary constriction.

When the speed of a motor vehicle doubles, the amount of kinetic energy: Select one: A. quadruples. B. is not affected. C. triples. D. doubles.

A. quadruples.

When caring for a patient with an open facial injury, the EMT's immediate priority should be to: Select one: A. wear gloves and facial protection. B. consider the mechanism of injury. C. manually stabilize the patient's head. D. closely assess the patient's airway.

A. wear gloves and facial protection.

Force acting over a distance is the definition of: Select one: A. work. B. latent energy. C. kinetic energy. D. potential energy.

A. work.

has a GCS score of less than or equal to 13 with a mechanism attributed to trauma

According to the American College of Surgeons Committee on Trauma (ACS-COT), an adult trauma patient should be transported to the highest level of trauma center if he or she:

traffic conditions hamper the ability to get the patient to a trauma center by ground within the ideal time frame for the best clinical outcome

According to the Association of Air Medical Services, you should consider air medical transport of a trauma patient if:

Air bags decrease injury to all of the following EXCEPT: A. chest B. heart C. face D. head

B

Applying ice to and elevating an injured extremity are performed in order to: A. prevent further injury. B. reduce pain and swelling. C. enhance tissue circulation. D. maintain extremity perfusion.

B

Because of local tenderness and swelling, it is easy to confuse a nondisplaced or minimally displaced fracture at the knee with a: A. tendon injury. B. ligament injury. C. dislocation. D. fracture-dislocation.

B

Bone marrow produces: A. platelets. B. blood cells. C. lymphocytes. D. electrolytes.

B

During your assessment of a patient with a head injury, you note that he opens his eyes when you pinch his trapezius muscle, is mumbling, and has his arms curled in toward his chest. You should assign him a GCS score of: A. 9. B. 7. C. 8. D. 10.

B

hematemesis

Common signs and symptoms of a chest injury include all of the following, EXCEPT:

chest pressure.

Common signs and symptoms of an airway burn include all of the following, EXCEPT: hoarseness. chest pressure. singed nasal hair. soot around the mouth.

positive indicators of a fracture

Crepitus and false motion are:

A "hip" fracture is actually a fracture of the: A. pelvic girdle. B. femoral shaft. C. pubic symphysis. D. proximal femur.

D

Which of the following patients has experienced the MOST significant fall? A. A 5′0″ patient who fell 13′ B. A 5′9″ patient who fell 14′ C. A 4′6″ patient who fell 13′ D. A 4′8″ patient who fell 13′

C

While assessing a young male who was struck in the chest with a steel pipe, you note that his pulse is irregular. You should be MOST suspicious for: A. a lacerated coronary artery. B. underlying cardiac disease. C. bruising of the heart muscle. D. traumatic rupture of the aorta.

C

While en route to a major motor vehicle crash, an on-scene police officer advises you that a 6-year-old male who was riding in the front seat is involved. He further states that the child was only wearing a lap belt and that the air bag deployed. On the basis of this information, you should be MOST suspicious that the child has experienced: A. open abdominal trauma. B. lower extremity fractures. C. neck and facial injuries. D. blunt trauma to the head.

C

With regard to musculoskeletal injuries, the zone of injury is defined as the: A. area of obvious deformity over the site of impact. B. exact part of the bone or joint that was disrupted. C. area of soft-tissue damage surrounding the injury. D. part of the body that sustained secondary injury.

C

Which of the following injuries would MOST likely occur as a direct result of the third collision in a motor vehicle crash? Select one: A. Flail chest B. Forehead lacerations C. Aortic rupture D. Extremity fractures

C. Aortic rupture

Closed chest injuries are typically caused by _______. Select one: A. flying debris B. penetrating trauma C. blunt trauma D. high-velocity weapons

C. blunt trauma

If a person is standing near a building that explodes, which of the following injuries would he or she MOST likely experience as a result of the pressure wave? A. Impaled objects B. Fractured bones C. Severe burns D. Stomach rupture

D

The index of suspicion is MOST accurately defined as: Select one: A. the detection of less obvious life-threatening injuries. B. a predictable pattern that leads to serious injuries. C. your awareness and concern for potentially serious underlying injuries. D. the way in which traumatic injuries occur.

C. your awareness and concern for potentially serious underlying injuries.

Signs and symptoms associated with hip dislocation include all of the following EXCEPT: A. severe pain in the hip. B. lateral and posterior aspects of the hip region will be tender on palpation. C. being able to palpate the femoral head deep within the muscles of the buttock. D. decreased resistance to any movement of the joint.

D

Signs and symptoms of a dislocated joint include all of the following EXCEPT: A. marked deformity. B. tenderness on palpation. C. locked joint. D. ecchymosis.

D

Signs and symptoms of knee ligament injury include: A. swelling. B. point tenderness. C. joint effusion. D. all of the above.

D

The zone of injury includes the: A. adjacent nerves. B. adjacent blood vessels. C. surrounding soft tissue. D. all of the above.

D

When splinting an injury of the wrist, the hand should be placed: A. into a fist. B. in a straight position. C. in an extended position. D. in a functional position.

D

Which of the following would MOST likely result from the third collision in the "three-collision" effect that occurs during a high-speed, frontal impact motor vehicle crash? A. Extensive damage to the automobile B. Flail chest and lower extremity fractures C. Massive external trauma with severe bleeding D. Aortic rupture or compression injury to the brain

D

Which of the following would MOST likely occur as the direct result of the second collision in a motor vehicle crash? A. Caved-in passenger door B. Collapsed dashboard C. Intrathoracic hemorrhage D. Deformed steering wheel

D

When a hollow organ is punctured during a penetrating injury to the abdomen: Select one: A. it commonly protrudes through the injury site. B. it will bleed profusely and rapidly cause shock. C. the abdomen will become instantly distended. D. peritonitis may not develop for several hours.

D. peritonitis may not develop for several hours.

Crepitus and false motion are: Select one: A. only seen with open fractures. B. indicators of a severe sprain. C. most common with dislocations. D. positive indicators of a fracture.

D. positive indicators of a fracture.

Because the depth of an open abdominal wound is often difficult to determine: Select one: A. the abdomen must be vigorously palpated. B. the EMT must perform a thorough exam. C. vital signs should be monitored frequently. D. prompt transport to the hospital is essential.

D. prompt transport to the hospital is essential.

An open fracture is MOST accurately defined as a fracture in which: Select one: A. a large laceration overlies the fracture. B. bone ends protrude through the skin. C. a bullet shatters the underlying bone. D. the overlying skin is no longer intact.

D. the overlying skin is no longer intact.

Whether you are using a commercial device or a stick and triangular bandage as a tourniquet, it is important to remember that: Select one: A. bulky dressings should be securely applied over the tourniquet to further assist in controlling the bleeding. B. you should try to control the bleeding by applying pressure to a proximal arterial pressure point first. C. the tourniquet should be applied directly over a joint if possible because this provides better bleeding control. D. the tourniquet should only be removed at the hospital because bleeding may return if the tourniquet is released.

D. the tourniquet should only be removed at the hospital because bleeding may return if the tourniquet is released.

A 4-year-old female has a peanut lodged in the external auditory canal of her right ear. You should: Select one: A. thoroughly flush her ear with sterile saline. B. use tweezers to try to remove the object. C. remove the peanut with a cotton-tipped swab. D. transport her to the emergency department.

D. transport her to the emergency department.

The sebaceous glands produce sebum, a material that: Select one: A. pulls the hair erect when you are cold. B. facilitates shedding of the epidermis. C. discharges sweat onto the skin's surface. D. waterproofs the skin and keeps it supple.

D. waterproofs the skin and keeps it supple.

In older patients, the first indicator of nontraumatic internal bleeding may be: Select one: A. a low blood pressure. B. a heart rate over 120 beats/min. C. diaphoresis and pale skin. D. weakness or dizziness.

D. weakness or dizziness.

hemorrhage

Damage to solid organs typically leads to _______.

intracranial bleeding

Following a blunt injury to the head, a 22-year-old female is confused and complains of a severe headache and nausea. On the basis of these signs and symptoms, you should be MOST concerned with the possibility of:

14

Following a head injury, a 20-year-old female opens her eyes spontaneously, is confused, and obeys your commands to move her extremities. You should assign her a GCS score of:

engorged jugular veins

Following a stab wound to the left anterior chest, a 25-year-old male presents with a decreased level of consciousness and signs of shock. Which of the following additional assessment findings should increase your index of suspicion for a cardiac tamponade?

provide ventilation assistance with a BVM

Following blunt trauma to the chest, an 18-year-old female presents with respiratory distress, shallow breathing, and cyanosis. Her blood pressure is 80/50 mm Hg and her pulse is 130 beats/min and thready. You should:

work

Force acting over a distance is the definition of:

immobilization of the injury.

Functions of dressings and bandages include all of the following, EXCEPT: immobilization of the injury. prevention of contamination. protection from further injury. control of external hemorrhage.

the production of key antibodies.

Functions of the skin include all of the following, EXCEPT: maintenance of water balance. regulation of body temperature. the production of key antibodies. sending information to the brain

myocardial contusion

a rapid, irregular pulse following blunt trauma to the chest is MOST suggestive of a

hematuria

a sign if kidney damage following blunt trauma is

is commonly caused by blunt chest trauma.

a simple pneumothorax

paralysis of the intercoastal muscles

a spinal cord injury at the level of C7 would MOST likely result in:

hangings

Hyperextension injuries of the spine are MOST commonly the result of:

must increase his or her respiratory rate to maintain adequate minute volume

If a patient with a chest injury is only able to inhale small amounts of air per breath, he or she:

stomach rupture

If a person is standing near a building that explodes, which of the following injuries would he or she MOST likely experience as a result of the pressure wave?

traumatic aortic rupture

Immediate death from blunt chest trauma following a motor vehicle crash is MOST often the result of:

primary assessment

Immediately life-threatening chest injuries must be found and managed during the _______.

infection

In addition to external bleeding, the MOST significant risk that an open soft-tissue injury exposes a patient to is

an air embolism

In addition to severe bleeding, the MOST life-threatening complication associated with an open neck injury is:

involves physical injury to the brain tissue

In contrast to a cerebral concussion, a cerebral contusion:

carries with it a wide variety of virulent bacteria and viruses.

In contrast to animal bites, the bite of a human

tachypnea

Patients with chest injuries will often present with _______.

Best Verbal Response

Oriented conversation - 5 Confused orientation - 4 Inappropriate words - 3 Incomprehensible sounds - 2 None - 1

flail chest

Paradoxical motion is a sign of a _______.

thoracic and lumbar

Passengers who are seated in the rear of a vehicle and are wearing only lap belts have a higher incidence of injuries to the _____________ spine during a rear-end crash.

eye opening, verbal response, and motor repsonse

The Glasgow Coma Scale (GCS) is used to assess:

establish an adequate airway

The MOST important immediate treatment for patients with a head injury, regardless of severity, is to:

delaying transport of a critically injured patient

The MOST significant hazard associated with splinting is:

Eye Opening

Spontaneous - 4 In response to speech - 3 In response to pain - 2 None - 1

falls

What is the most common mechanism of injury in geriatric patients?

somatic

What part of the nervous system controls the body's voluntary activities?

door

When a driver is in a car equipped with an air bag, but is not wearing a seatbelt, he or she will MOST likely strike the __________ when the air bag deploys upon impact.

may rise as high as the nipple line.

When a person is lying supine at the end of exhalation, the diaphragm:

pupillary constriction

When activated, the sympathetic nervous system produces all of the following effects, EXCEPT:

determine if the strength in all extremities in equal

When assessing a conscious patient with an MOI that suggests spinal injury, you should:

primary blast injuries are the most easily overlooked

When assessing a patient who experienced a blast injury, it is important to remember that:

rupture of the tympanic membrane following diffuse impact to the head

When assessing a patient with a head injury, you note the presence of thin, bloody fluid draining from his right ear. This indicates:

osteoprorosis can cause a fracture as a result of a fall from a standing position

When assessing an elderly patient who fell, it is important to remember that:

brush away the chemical before flushing with water

When caring for a patient whose arm is covered with a dry chemical, you should:

carefully assess the patient for breathing problems

When caring for a patient with a possible fracture of the scapula, the EMT should:

level of consciousness

When caring for a patient with a possible head injury, it is MOST important to monitor the patient's:

avoid excessive pressure when applying the bandage

When controlling bleeding from a scalp laceration with a suspected underlying skull fracture, you should:

avoid speculation and document only factual data

When documenting a call involving a female patient who was sexually assaulted, the EMT should

appx the speed of the vehicle that struck the pedistrian

When evaluating the mechanism of injury of a car-versus-pedestrian collision, you should first:

the patient has been completely secured to the backboard

When immobilizing a trauma patient's spine, the EMT manually stabilizing the head should not let go until:

jaw-thrust maneuver

When opening the airway of a patient with a suspected spinal injury, you should use the:

head

When placing a patient onto a long backboard, the EMT at the patient's _________ is in charge of all patient movements.

quadruples

When the speed of a motor vehicle doubles, the amount of kinetic energy:

avoid the use of creams, lotions, or antiseptics.

When treating a partial-thickness burn, you should:

ears

Which of the following areas of the body has the thinnest skin?

Level I or Level II trauma center

Which of the following destinations is most appropriate for a 41-year-old male patient who was involved in a rollover motor vehicle collision and is unconscious and unresponsive, assuming that travel times to each is equal?

delopyment of the air bag

Which of the following findings would be LEAST suggestive of the presence of high-energy trauma?

aortic rupture

Which of the following injuries would MOST likely occur as a direct result of the third collision in a motor vehicle crash?

rapid transport to a trauma center

Which of the following interventions is the MOST critical to the outcome of a patient with multisystem trauma?

D. Partial-thickness burn to 20% of the BSA

Which of the following is a severe burn in a 65-year-old patient? A. Second-degree burn covering 10% of the BSA B. Full-thickness burn to 1% of the BSA C. Superficial burn to 30% of the BSA D. Partial-thickness burn to 20% of the BSA

Pain upon passive movement of the injury site

Which of the following is a sign of compartment syndrome

aortic rupture

Which of the following is most likely to cause immediate death?

displaced fracture

Which of the following musculoskeletal injuries would MOST likely result in deformity?

pelvic fracture with hypotension

Which of the following musculoskeletal injuries would pose the greatest threat to a patient's life?

abrasion

Which of the following open soft-tissue injuries is limited to the superficial layer of the skin and results in the least amount of blood loss?

The immune system releases histamines, which cause vasodilation and increased capillary permeability, resulting in local redness and swelling

Which of the following processes occurs during the inflammation phase of the healing process?

linear

Which of the following skull fractures would be the LEAST likely to present with palpable deformity or other outward signs?

the potential for airway swelling

You and your partner arrive at the scene of a house fire where firefighters have rescued a 50-year-old male from his burning house. The patient has superficial and partial-thickness burns to his face and chest. His nasal hairs are singed and he is coughing up sooty sputum. You should be MOST concerned with:

the fact that her clinical signs could indicate that she is bleeding internally.

You are assessing a 30-year-old woman with multiple large bruises to her chest and abdomen that she experienced during an assault. She is conscious but restless, and her skin is cool and pale. You should be MOST concerned with:

bacterial menigitis

You are assessing a man who has a head injury and note that cerebrospinal fluid is leaking from his ear. You should recognize that this patient is at risk for:

law enforcement is at the scene

You are dispatched to a residence for a young female who was kicked in the abdomen. While en route to the scene, you should ask the dispatcher if:

compression of the aorta and vena cava

You are transporting a stable patient with a possible pneumothorax. The patient is receiving high-flow oxygen and has an oxygen saturation of 95%. During your reassessment, you find that the patient is now confused, hypotensive, and profusely diaphoretic. What is MOST likely causing this patient's deterioration?

laceration of the aorta

You arrive at the scene of a major motor vehicle crash. The patient, a 50-year-old female, was removed from her vehicle prior to your arrival. Bystanders who removed her state that she was not wearing a seatbelt. The patient is unresponsive, tachycardic, and diaphoretic. Your assessment reveals bilaterally clear and equal breath sounds, a midline trachea, and collapsed jugular veins. You should be MOST suspicious that this patient has experienced a:

assess distal circulation and readjust the bandage as needed.

You have applied a dressing and roller-gauze bandage to a laceration on the arm of a young female. During transport, she begins to complain of numbness and tingling in her hand. You should:

partially remove the dressing

You have sealed the open chest wound of a 40-year-old male who was stabbed in the anterior chest. Your reassessment reveals that he is experiencing increasing respiratory distress and tachycardia, and is developing cyanosis. You should:

a segment of the chest wall is detached from the thoracic cage

a flail chest occurs when:

joint

a junction where two bones come into contact. stabilize in key areas by ligaments

an assessment more focused on the chief complaint while still assessing the patient as a whole

a nonsignificant MOI should recieve

secondary assessment including rapid examination of the entire body

a patient who has sustained a significant MOI and is considered to be in serious or critical condition should recieve

penetrate the lung and diaphragm and injure the liver or stomach

a penetrating chest injury has the potential to

high-flow oxygen or ventilation with BVM

all patients with chest injuries should recieve

air enters the pleural space from outside the body

an open pneumothorax occurs when:

Skeletal or voluntary muscle

attaches to bone and forms the major muscle mass of the body. this muscle contains veins, arteries and nerves

depth and extent

burns are classified primarily by __ and __ of the burn injury and the body area involved.

open or closed

chest injuries are classifed as

cerebellum

coordination of balance and body movement is controlled by the:

lungs

hemoptysis indicates damage to the

have general surgeons who are in house 24 hr a day

in contrast to a Level III trauma center, a level I trauma center must

identify contact points and predict potential injuries

evaluation of the interior of a crashed motor vehicle during extrication will allow the EMT to

the bullet may tumble or ricochet within the body

interal injuries caused by gunshot wounds are difficult to predict because

pleurisy

irritation or damage to the pleural surfaces that causes sharp chest pain during inhalation is called:

Traumatic brain injury

is a severe head injury that can be life threat or leave the patient with life altering injuries

sprain

is a stretching injury to the liagments around a joint

head injury

is a traumatic injury to the head that may result in injury to soft tissue, bony structures, or the brain

keeping the injury moist and warm

management of esvisceration injuries includes

pneumothorax

may progress to a tension __ and cause cardiac arrest

blunt trauma

may result in fractures to the ribs and the sternum

urinary

most of the ___ system lies behind the digestive tract

frontal(head-on), lateral (t-bone), rear-end, rotational(spins) and rollovers

motor vehicle crashes are classified traditionally as

life-threatening hemorrage

must be addressed immediately during the primary assessment even before airway or breathing concerns

closed injuries

often the result of blunt force trauma

breathe rapidly and shallowly

patients with rib fractures will commonly

primary blast injuries, secondary blast injuries, tertiary blast injuries, and or miscellaneous blast injuries

people who are injured in explosions may have injuries that are classified as

skin

protects the body by keeping pathogens out and water in and assisting in body temp regulation

closed injury-

soft tissure damage occurs beneath the skin or mucous membrane, but the surface remains intact

peripheral

the ___ nervous system consists of 31 pairs of spinal nerves and 12 pairs of cranial nerves

increases the risk of internal injury

the pliability of the pediatric rib cage

burns

the soft tissues recieves more energy than it can absorb without injury, the source of this energy can be thermal, toxic chemicals, electricity or radiaion

cavitation

the temporary wound channel caused by high-velocity weapons is called

inserting a needle through the rib cage into the pleural space

the treatment for relieving a tension pneumothorax involves

open injuries

there is abreak in the surface of the skin or the mucous membrane, exposing deeper tissue to potential contaimination

myocardial contusion -bruising of the heart muscle after traumatic injury

this condition may have the same signs and symptons as a heart attack, including an irregular pulse, this injuryis an injury to the heart muscle from trauma not from a heart attack

potential energy, kinetic energy and work

three concepts of energy are typically associated with injury

flail chest segment postive-pressure ventilation

three or more ribs broken in two or more places what ventilation may be particularly important for a patient with a flail chest that compromises ventilation

establish and adequate airway, control bleeding, and reassess the patient's baseline level of consciousness

treatment for a patient with a head injury include

maintain the airway while keeping the spine properly aligned, assess respirations and give supplemental oxygen

treatment for a patient with a spinal injury include

1) collisions of vehicle against some type of object 2)collisions of passenger against the interior of the vehicle 3)collisions of the passengers internal organs against the solid structures of the body

what are the 3 types of collisions

closed injuries, open injuries, and burns

what are three types of soft tissue injuries

Rest, Ice, Compression, Elevation, and Splinting

what does RICES stand for

pulmonary contusion-bruising of or injury to lung tissue after traumatic injury

what may interfere with oxygen exchange in the lung tissue

open pneumothorax

when a penetrating injury creates a hole in the chest wall, you may hear a sucking sound as the patient inhales this is called

signs and symptons of shock

when assessing a patinet with a hemothorax, you will MOST likely find

MOI

when assessing the interior of a crash motor vehicle for damage, you are gathering information regarding the

cardiac tamponade

when blood collects in the space between the pericardial sac and the heart. this condition results in pressure building up inside the pericardial sac until the heart cannot pump effectively; cardiac arrest may occur quickly

place padding under the child's shoulder as needed

when immoblizing a child on a long backboard, you should

avoid giving oxygen under positive pressure

when treating a patient who experienced a pulmonary blast injury, you should

irregular rate, pattern, and volume of breathing with intermittent periods of apnea

which of the following breathing pattersn is MOST indicative of increased intracranial pressure?

urinary bladder

which of the following organs is at MOST risk for injury as the result of a pelvic fracture

a 4'6 patient who fell 13 feet

which of the following patients has experienced the MOST significant fall? a 4′8″ patient who fell 12 feet a 5′0″ patient who fell 13 feet a 4′6″ patient who fell 13 feet a 5′9″ patient who fell 14 feet

the speed of a bullet has the greatest impact on the injury producted

which of the following statements regarding gunshot wounds is correct

the area of injury is usually close to the path the object took through the body

which of the following statements regarding low energy penetrating injuries is correct

hypoxia and hypotension are the two most common causes of secondary brain injury

which of the following statements regarding secondary brain injury is correct

slow

you should be MOST suspicious that a patient has experienced a significant head injury if his or her pulse is:

CONTUSION

A closed soft-tissue injury characterized by swelling and ecchymosis is called a(n):

A 12-year-old male jumped approximately 12′ from a tree and landed on his feet. He complains of pain to his lower back. What injury mechanism is MOST likely responsible for his back pain? A. Energy transmission to the spine B. Secondary fall after the initial impact C. Direct trauma to the spinal column D. Lateral impact to the spine

A

A 21-year-old male was thrown over the handlebars of his motorcycle when he rear-ended a car that was stopped at a red light. He was wearing a helmet, which he removed prior to your arrival. He is conscious, but restless, and has closed deformities to both of his femurs. His skin is pale, his heart rate is rapid and weak, and his respirations are rapid and shallow. In addition to applying high-flow oxygen and protecting his spine, you should: A. bind his legs together on the backboard, keep him warm, and transport without delay. B. apply traction splints to both of his legs, keep him warm, and transport without delay. C. splint each of his deformed femurs with long board splints and transport immediately. D. apply the PASG to stabilize his femurs and transport at once.

A

A 31-year-old male fell and landed on his left elbow. Your assessment reveals that the elbow is grossly deformed, his forearm is cool and pale, and the distal pulse on the side of the injury is barely palpable. His vital signs are stable and he denies any other injuries. Your transport time to the closest appropriate hospital is approximately 15 minutes. You should: A. splint the elbow in the position found and transport. B. gently manipulate the elbow to improve circulation. C. gently straighten the elbow and apply an air splint. D. apply a heat pack to the elbow to reduce swelling.

A

A _______ is a stretching or tearing of the muscle. A. strain B. sprain C. torn ligament D. split

A

A comprehensive regional resource capable of providing every aspect of trauma care from prevention through rehabilitation is the definition of a _____ trauma center. A. Level I B Level II C. Level III D. Level IV

A

A(n) _______ fracture is an incomplete fracture that passes only partway through the shaft of a bone but many still cause severe angulation. A. greenstick B. comminuted C. pathologic D. epiphyseal

A

A(n) _______ fractures the bone at the point of impact. A. direct blow B. indirect force C. twisting force D. high-energy injury

A

According to the Association of Air Medical Services (AAMS), you should consider air medical transport of a trauma patient if: A. traffic conditions hamper the ability to get the patient to a trauma center by ground within the ideal time frame for the best clinical outcome. B. he or she was involved in a motor vehicle crash in which another occupant in the same vehicle was killed, even if your patient's injuries are minor. C. ground transport will take your ambulance out of service for an extended period of time, regardless of the severity of the patient's injuries. D. the patient requires advanced life support care and stabilization, and the nearest ALS-ground ambulance is more than 5 to 10 minutes away.

A

Air bags are designed to: A. decrease the severity of deceleration injuries. B. prevent the driver from sustaining head trauma. C. be used with or without a shoulder harness. D. prevent a second collision inside the car.

A

An unrestrained driver collided with a bridge pillar. Upon inspection of the interior of his vehicle, you note that the lower dashboard is crushed. During your assessment of the patient, you will MOST likely encounter: A. trauma to the pelvis. B. blunt abdominal trauma. C. a severe closed head injury. D. penetrating thoracic trauma.

A

Blood in the urine is known as: A. hematuria. B. hemotysis. C. hematocrit. D. hemoglobin.

A

Damage to the body that resulted from a pressure wave generated by an explosion is found in what type of blast: A. primary B. secondary C. tertiary D. miscellaneous

A

Deformity caused by a fracture would MOST likely be masked by: A. swelling. B. guarding. C. crepitus. D. ecchymosis.

A

Femoral shaft fractures can result in up to _____ mL of internal blood loss. A. 1,000 B. 1,500 C. 2,000 D. 4,000

A

Following a blunt injury to the head, a 22-year-old female is confused and complains of a severe headache and nausea. On the basis of these signs and symptoms, you should be MOST concerned with the possibility of: A. intracranial bleeding. B. a fracture of the skull. C. spinal cord injury. D. airway compromise.

A

Fracture of the tibia and fibula are often associated with _______ as a result of the distorted positions of the limb following injury. A. vascular injury B. muscular injury C. tendon injury D. ligament injury

A

In a motor vehicle collision, as the passenger's head hits the windshield, the brain continues to move forward until it strikes the inside of the skull, resulting in a _____ injury. A. compression B. laceration C. lateral D. motion

A

In moving joints, the ends of the bones are covered with: A. articular cartilage. B. synovial tendons. C. muscular fascia. D. gliding cartilage.

A

Of the following musculoskeletal injuries, which is considered to be the LEAST severe? A. nondisplaced pelvic fracture B. open fractures of a long bone C. an amputation of an extremity D. multiple closed long bone fractures

A

To effectively immobilize a fractured clavicle, you should apply a(n): A. sling and swathe. B. air splint over the entire arm. C. rigid splint to the upper arm, then a sling. D. traction splint to the arm of the injured side.

A

When assessing an elderly patient who fell, it is important to remember that: A. osteoporosis can cause a fracture that may have resulted in the fall. B. elderly patients who fall usually have a secondary head injury. C. bilateral hip fractures usually occur when an elderly person falls. D. any fall in the elderly is considered to be high-energy trauma

A

When caring for a patient with a possible fracture of the scapula, the EMT should: A. carefully assess the patient for breathing problems. B. apply rigid board splints across the chest and back. C. assume that minimal force was applied to the back. D. recognize that scapular fractures are life threatening.

A

When caring for an occupant inside a motor vehicle equipped with an air bag that did not deploy upon impact, it is MOST important to: A. remember that it could still deploy and seriously injure you. B. suspect that the patient may have experienced serious injuries. C. realize that the air bag malfunctioned at the time of impact. D. recognize that the force of impact was most likely not severe.

A

Which of the following findings would be LEAST suggestive of the presence of high-energy trauma? A. Deployment of the air bag B. Steering wheel deformity C. Intrusion into the vehicle D. Dismounted seats

A

Which of the following interventions is the MOST critical to the outcome of a patient with multisystem trauma? A. Rapid transport to a trauma center B. Elevation of the lower extremities C. Intravenous fluid administration D. Early administration of oxygen

A

Which of the following scenarios is an example of a direct injury? A. A passenger fractures her patella after it strikes the dashboard. B. A skier dislocates his knee after a twisting injury to the ankle. C. A person lands on his or her feet and fractures the lumbar spine. D. A child dislocates his elbow after falling on his outstretched arm.

A

Which of the following statements regarding compartment syndrome is correct? A. Compartment syndrome typically develops within 6 to 12 hours after an injury. B. Compartment syndrome occurs because of increased pressure within the bone cavity. C. In most cases, compartment syndrome develops within a few minutes after an injury. D. Most cases of compartment syndrome occur following a severe fracture of the pelvis.

A

Which of the following statements regarding the treatment of an amputation is FALSE? A. You should sever any partial amputation, because this will aid in the reattachment process. B. In some areas, wrapping the amputated part in a dry sterile dressing is appropriate. C. In some areas, wrapping the amputated part in dressings moistened with sterile saline is appropriate. D. After wrapping the amputated part, place it in a plastic bag.

A

While transporting a patient, you continue to recheck the splint you applied. You know that improperly applying a splint can cause all of the following EXCEPT: A. increase of distal circulation if the splint is too tight. B. delay in transport of a patient with a life-threatening injury. C. aggravation of the distal circulation. D. compression of nerves, tissues, and blood vessels.

A

Whiplash injuries are MOST common following _________ impacts. A. rear-end B. rollover C. frontal D. lateral

A

You receive a call to a local gymnasium for a basketball player with a dislocated shoulder. Upon arrival, you find the patient, a 17-year-old male, sitting on the ground. He is holding his left arm in a fixed position away from his body. There is an obvious anterior bulge to the area of injury. You should: A. assess distal pulse, motor, and sensory functions. B. gently attempt to move his arm toward his body. C. place a pillow under his arm and apply a swathe. D. flex his arm at the elbow and then apply a sling.

A

You respond to a 19-year-old woman who was involved in a motor vehicle collision. She is alert and oriented. Her airway is open, and respirations are 18 breaths/min and unlabored. Pulse is 94 beats/min and is strong and regular. Distal pulses are present. Her upper arm has obvious deformity. You splint the upper arm. You know that splinting will do all of the following EXCEPT: A. prevent the need for surgery. B. reduce shock. C. minimize compromised circulation. D. reduce pain.

A

You respond to a 19-year-old woman who was kicked in the leg by a horse. She is alert and oriented. Respirations are 20 breaths/min, regular and unlabored. Pulse is 110 beats/min and regular. Distal pulses are present. She has point tenderness at the site of the injury. You should compare the limb to: A. the opposite uninjured limb. B. one of your limbs or one of your partner's limbs. C. an injury chart. D. none of the above.

A

Your 24-year-old patient fell off a balance beam and landed on his arm. He is complaining of pain in the upper arm, and there is obvious swelling. You know that swelling is a sign of: A. bleeding. B. laceration. C. a locked joint. D. compartment syndrome.

A

energy transmission to the spine

A 12-year-old male jumped approximately 12 feet from a tree and landed on his feet. He complains of pain to his lower back. What injury mechanism is MOST likely responsible for his back pain?

stabilize her entire spine

A 15-year-old female was struck by a small car while riding her bicycle. She was wearing a helmet and was thrown to the ground, striking her head. In addition to managing problems associated with airway, breathing, and circulation, it is MOST important for you to:

keep her knee flexed and secure it with padded board splints

A 17-year-old female dislocated her patella while playing soccer. Her knee is flexed and she complains of severe pain. You should:

Ventricular fibrillation when the impact occurred during a critical portion of the cardiac cycle

A 19-year-old male is unresponsive, apneic, and pulseless after being struck in the center of the chest with a softball. Based on the mechanism of injury, what MOST likely occurred?

assume that the arrow injured an internal organ

A 20-year-old male was accidentally shot in the right upper abdominal quadrant with an arrow during an archery contest. Prior to your arrival, the patient removed the arrow. Your assessment reveals that he is conscious and alert with stable vital signs. The entrance wound is bleeding minimally and appears to be superficial. You should:

18%

A 21-year-old male was working in an auto repair shop and sustained radiator burns to the anterior aspect of both arms and to his anterior chest. According to the rule of nines, this patient has burns that cover _____ of his BSA.

stablize her head and perform a primary assessment

A 22-year-old female was ejected from her car after striking a tree head-on. As you approach her, you note obvious closed deformities to both of her femurs. She is not moving and does not appear to be conscious. You should:

prepare for immediate transport

A 28-year-old male was struck in the chest with a baseball bat during an altercation. He is conscious and alert and complains of severe chest pain. Your assessment reveals a large area of ecchymosis over the sternum and a rapid, irregular pulse. In addition to providing supplemental oxygen, you should:

compartment syndrome.

A 30-year-old male experienced a crushing injury when his forearm was trapped between the back of a truck and a loading dock. Upon your arrival, the man's arm has been freed. Your assessment reveals that his arm is obviously deformed and swollen and is cold and pale. Further assessment reveals an absent radial pulse. You should be MOST concerned that this patient has

potiental airway compromise

A 30-year-old male sustained a stab wound to the neck when he was attacked outside a nightclub. During your assessment, you should be MOST alert for:

Cover it with moist, sterile gauze and secure with an occlusive dressing

A 33-year-old male sustained an abdominal evisceration to the left lower quadrant of his abdomen after he was cut with a large knife. After appropriately managing his ABCs and assessing him for other life-threatening injuries, how you should care for his wound?

administer high-flow oxygen

A 39-year-old male was struck in the head by a baseball during a game. He is confused and has slurred speech. He has a large hematoma in the center of his forehead and cannot remember the events preceding the injury. After manually stabilizing his head and assessing his airway, you should:

giving him small sips of plain water.

A 40-year-old male presents with severe abdominal pain following blunt trauma. He is diaphoretic, intensely thirsty, and has a weak and rapid pulse. Appropriate treatment for this patient includes all of the following, EXCEPT: covering him with a warm blanket. giving him small sips of plain water. promptly transporting to the hospital. administering supplemental oxygen.

30%

A 5-year-old female pulled a pot of boiling water from the stove. She has superficial and partial-thickness burns to her head, face, and anterior trunk. What percentage of her body surface area has been burned?

control the bleeding and cover the wound with a sterile dressing

A 54-year-old male accidentally shot himself in the leg while cleaning his gun. Your assessment reveals a small entrance wound to the medial aspect of his right leg. The exit wound is on the opposite side of the leg and is actively bleeding. The patient complains of numbness and tingling in his right foot. You should:

replace the avulsed flap to its original position and cover it with a sterile dressing

A 56-year-old male has an incomplete avulsion to his right forearm. After controlling any bleeding from the wound, you should:

have you experienced recent abdominal trauma

A 66-year-old male presents with dark red rectal bleeding and abdominal pain. He is conscious and alert; however, his skin is cool and clammy and his pulse is rapid. Further assessment reveals that his blood pressure is 112/60 mm Hg. Which of the following questions would be MOST pertinent to ask him?

superficial burn.

A burn that is characterized by redness and pain is classified as a:

control the bleeding, begin CPR, stabilize the steel rod, immobilize his spine, and transport immediately.

A construction worker fell approximately 30 feet and landed in a pile of steel rods. Your assessment reveals that he is pulseless and apneic and has a 10-inch steel rod impaled in his left leg. You should:

blunt injury to the kidney

A football player was struck by another player in the right flank area just below the posterior rib cage. He complains of severe pain and point tenderness to the area. Your assessment reveals that there is a small amount of blood in his underwear. You should be MOST suspicious for:

break in the continuity of the bone

A fracture is MOST accurately defined as a(n):

large blood vessels beneath the skin are damaged.

A hematoma develops when

compression of the head against the roof

A small compact car was involved in a rollover crash. As you are approaching the vehicle, you note that the roof is significantly collapsed. The patient, a 29-year-old male, is complaining of severe pain in his neck and to the top of his head as well as numbness and tingling in his extremities. Witnesses who removed the patient from the vehicle state that he was wearing his seatbelt. What injury mechanism is MOST likely responsible for this patient's condition?

distal humerus

A supracondylar or intercondylar fracture is also known as a fracture of the

it interferes with your assessment of the airway

A tight-fitting motorcycle helmet should be left in place unless

assess for an exit wound

A young male sustained a gunshot wound to the abdomen during an altercation. As your partner is assessing and managing his airway, you should control the obvious bleeding and then:

Which of the following destinations is most appropriate for a 41-year-old male patient who was involved in a rollover motor vehicle collision and is unconscious and unresponsive, assuming that travel times to each is equal? Select one: A. A Level I or Level II trauma center B. Only a Level I trauma center C. A Level IV or Level III trauma center D. Any designated trauma center is acceptable.

A. A Level I or Level II trauma center

Which of the following splinting devices would be MOST appropriate to use for a patient who has an open fracture of the forearm with external bleeding? Select one: A. Air splint B. Cardboard splint C. Vacuum splint D. Sling and swathe

A. Air splint

Which of the following statements regarding compartment syndrome is correct? Select one: A. Compartment syndrome typically develops within 6 to 12 hours after an injury. B. Compartment syndrome occurs because of increased pressure within the bone cavity. C. In most cases, compartment syndrome develops within a few minutes after an injury. D. Most cases of compartment syndrome occur following a severe fracture of the pelvis.

A. Compartment syndrome typically develops within 6 to 12 hours after an injury.

Which of the following musculoskeletal injuries would MOST likely result in deformity? Select one: A. Displaced fracture B. Moderate sprain C. Hairline fracture D. Severe strain

A. Displaced fracture

Which of the following areas of the body has the thinnest skin? Select one: A. Ears B. Scalp C. Back D. Soles of the feet

A. Ears

Which of the following is true regarding injury to the kidneys? Select one: A. Injury to the kidneys usually indicates injury to other organs. B. Only minimal force is needed to damage the kidneys. C. The kidneys are not well protected. D. Kidney injuries are rarely caused by blunt trauma.

A. Injury to the kidneys usually indicates injury to other organs.

Which of the following statements regarding the vitreous humor is correct? Select one: A. It is a clear, jellylike fluid near the back of the eye that cannot be replaced if it is lost. B. It is a clear fluid that is produced by the lacrimal glands and cannot be replaced if it is lost. C. It is a clear, watery fluid that is located in front of the lens and can be replaced if it is lost. D. It is a clear, watery fluid surrounding the eye and can be replaced if it is lost.

A. It is a clear, jellylike fluid near the back of the eye that cannot be replaced if it is lost.

Which portion of the blood carries oxygen to and wastes away from body tissues? Select one: A. Red blood cells B. Platelets C. Plasma D. White blood cells

A. Red blood cells

What part of the nervous system controls the body's voluntary activities? Select one: A. Somatic B. Autonomic C. Sensory D. Central

A. Somatic

Which of the following findings would be the MOST significant when assessing a patient with possible internal bleeding? Select one: A. The patient takes rivaroxaban (Xeralto). B. The patient has a history of hypertension. C. The patient has not eaten in 24 hours. D. The patient had a stroke 5 years prior.

A. The patient takes rivaroxaban (Xeralto).

Following blunt trauma to the face, a 21-year-old male complains of a severe headache and decreased ability to move his eyes. This patient's clinical presentation is MOST consistent with: Select one: A. a blowout fracture. B. optic vessel compression. C. a lacerated globe. D. a ruptured eyeball.

A. a blowout fracture.

A flail chest occurs when: Select one: A. a segment of the chest wall is detached from the thoracic cage. B. multiple ribs are fractured on both sides of the thoracic cage. C. more than three ribs are fractured on the same side of the chest. D. a segment of fractured ribs bulges during the inhalation phase.

A. a segment of the chest wall is detached from the thoracic cage.

While jogging, a 19-year-old male experienced an acute onset of shortness of breath and pleuritic chest pain. He is conscious and alert with stable vital signs. Your assessment reveals that he has diminished breath sounds over the left side of the chest. You should: Select one: A. administer oxygen and transport to the hospital. B. immediately perform a rapid head-to-toe exam. C. circumferentially tape a dressing around his chest. D. recognize that he needs a needle decompression.

A. administer oxygen and transport to the hospital.

In addition to severe bleeding, the MOST life-threatening complication associated with an open neck injury is: Select one: A. an air embolism. B. a spinal fracture. C. an ischemic stroke. D. nerve fiber damage.

A. an air embolism.

A fracture is MOST accurately defined as a(n): Select one: A. break in the continuity of the bone. B. abnormality in the structure of a bone. C. total loss of function in a bone. D. disruption in the midshaft of a bone.

A. break in the continuity of the bone.

When caring for a patient with a possible fracture of the scapula, the EMT should: Select one: B. apply rigid board splints across the chest and back. C. assume that minimal force was applied to the back. D. recognize that scapular fractures are life threatening.

A. carefully assess the patient for breathing problems.

A 54-year-old male accidentally shot himself in the leg while cleaning his gun. Your assessment reveals a small entrance wound to the medial aspect of his right leg. The exit wound is on the opposite side of the leg and is actively bleeding. The patient complains of numbness and tingling in his right foot. You should: Select one: A. control the bleeding and cover the wound with a sterile dressing. B. assess distal pulses as well as sensory and motor functions. C. gently manipulate the injured leg until the numbness dissipates. D. manually stabilize the leg above and below the site of injury.

A. control the bleeding and cover the wound with a sterile dressing.

A closed soft-tissue injury characterized by swelling and ecchymosis is called a(n): Select one: A. contusion. B. crush injury. C. abrasion. D. hematoma.

A. contusion.

Bleeding from soft-tissue injuries to the face is MOST effectively controlled with: Select one: A. direct pressure using dry, sterile dressings. B. digital pressure to an adjacent pulse point. C. ice packs and elevation of the patient's head. D. pressure dressings and chemical ice packs.

A. direct pressure using dry, sterile dressings.

When a patient stiffens the muscles of the abdomen, it is known as _______. Select one: A. guarding. B. distention. C. crepitus. D. instability.

A. guarding.

In contrast to a cerebral concussion, a cerebral contusion: Select one: A. involves physical injury to the brain tissue. B. does not cause pressure within the skull. C. usually does not cause a loss of consciousness. D. results from a laceration to the brain tissue.

A. involves physical injury to the brain tissue.

A 17-year-old female dislocated her patella while playing soccer. Her knee is flexed and she complains of severe pain. You should: Select one: A. keep her knee flexed and secure it with padded board splints. B. flex her knee slightly more and assess for distal circulation. C. gently straighten her knee and apply a padded board splint. D. make one attempt to return the patella to its normal position.

A. keep her knee flexed and secure it with padded board splints.

When a person is lying supine at the end of exhalation, the diaphragm: Select one: A. may rise as high as the nipple line. B. is less prone to penetrating trauma. C. descends below the level of the navel. D. contracts and flattens inferiorly.

A. may rise as high as the nipple line.

Circulation of blood within an organ or tissue in adequate amounts to meet the cells' oxygen, nutritional, and waste-removal needs is termed _______. Select one: A. perfusion B. hypoperfusion C. coagulation D. hemorrhage

A. perfusion

When assessing a patient with a head injury, you note the presence of thin, bloody fluid draining from his right ear. This indicates: Select one: A. rupture of the tympanic membrane following diffuse impact to the head. B. fractures to the internal structures of the ear following direct trauma. C. a linear skull fracture and a significant increase in intracranial pressure. D. significant pressure and bleeding in between the skull and dura mater.

A. rupture of the tympanic membrane following diffuse impact to the head.

Following blunt trauma to the abdomen, a 21-year-old female complains of diffuse abdominal pain and pain to the left shoulder. Your assessment reveals that her abdomen is distended and tender to palpation. On the basis of these findings, you should be MOST suspicious of injury to the: Select one: A. spleen. B. gallbladder. C. pancreas. D. liver.

A. spleen.

A 22-year-old female was ejected from her car after striking a tree head-on. As you approach her, you note obvious closed deformities to both of her femurs. She is not moving and does not appear to be conscious. You should: Select one: A. stabilize her head and perform a primary assessment. B. administer oxygen and perform a rapid assessment. C. apply manual stabilization to both of her femurs. D. assess for a carotid pulse and assist her ventilations.

A. stabilize her head and perform a primary assessment.

During your secondary assessment of a 30-year-old male who fell 25 feet, you note crepitus when palpating his pelvis. Your partner advises you that the patient's blood pressure is 80/50 mm Hg and his heart rate is 120 beats/min and weak. After completing your assessment, you should: Select one: A. stabilize the pelvis with a pelvic binder and protect the spine. B. log roll the patient onto a long backboard and transport at once. C. perform a focused physical exam with emphasis on the pelvis. D. defer spinal immobilization and transport to a trauma center.

A. stabilize the pelvis with a pelvic binder and protect the spine.

Signs of a pulmonary blast injury include: A. multiple rib fractures. B. coughing up blood. C. an irregular pulse. D. vomiting blood.

B

The severity of bleeding should be based on all of the following findings, EXCEPT: Select one: A. systolic blood pressure. B. poor general appearance. C. the mechanism of injury. D. clinical signs and symptoms.

A. systolic blood pressure.

The ideal procedure for moving an injured patient from the ground to a backboard is: Select one: A. the four-person log roll. B. the direct patient carry. C. the use of a scoop stretcher. D. the clothes drag.

A. the four-person log roll.

When immobilizing a trauma patient's spine, the EMT manually stabilizing the head should not let go until: Select one: A. the patient has been completely secured to the backboard. B. an appropriately sized cervical collar has been applied. C. the patient has been secured to the ambulance stretcher. D. the head has been stabilized with lateral immobilization.

A. the patient has been completely secured to the backboard.

In general, musculoskeletal injuries should be splinted before moving the patient unless: Select one: A. the patient is clinically unstable. B. transport time is less than 15 minutes. C. deformity and swelling are present. D. the patient is in severe pain.

A. the patient is clinically unstable.

Children are often "belly breathers" because _______. Select one: A. their intercostal muscles are not developed B. they are routinely hypoxic C. their diaphragm is not functional D. they are consciously controlling ventilations

A. their intercostal muscles are not developed

Skeletal muscle is also referred to as __________ muscle. A. smooth B. striated C. connective D. involuntary

B

report the incident to the appropriate authorities.

An 8-year-old male was bitten by a stray dog. He has a large laceration to the back of his left hand, which your partner covered with a sterile dressing and bandage. In addition to transporting the child to the hospital, you should:

occurs when organs protrude through an open wound.

An abdominal evisceration:

a rapid deterioration of neurologic signs.

An indicator of an expanding intracranial hematoma or rapidly progressing brain swelling is:

the overlying skin is no longer in tact

An open fracture is MOST accurately defined as a fracture in which:

apply direct pressure to the wound

An unresponsive trauma patient has a large open abdominal wound with massive external bleeding. You should

Skeletal muscle is attached to the bone by tough, ropelike fibrous structures called: A. fascia. B. tendons. C. cartilage. D. ligaments.

B

Smooth muscle is found in the: A. back. B. blood vessels. C. heart. D. all of the above.

B

Which of the following structures is regulated by smooth muscle? A. heart B. skeleton C. blood vessels D. diaphragm

C

open his airway and assess his breathing status.

As you approach a young male who was involved in an industrial accident, you note that his eyes are closed and that he is not moving. You can see several large contusions to his arms, a laceration to his forehead with minimal bleeding, and a closed deformity to his right leg. You should:

A 40-year-old unrestrained female impacted the steering wheel of her vehicle with her chest when she hit a tree while traveling at 45 mph. She is conscious and alert, but is experiencing significant chest pain and shortness of breath. Which of the following injuries did this patient likely NOT experience? A. Cardiac contusion B. Head injury C. Multiple rib fractures D. Pulmonary contusion

B

A 45-year-old female was the unrestrained passenger of a small car that rear-ended another vehicle at a moderate rate of speed. She is conscious and alert, but complains of pain to both of her knees. There is visible damage to the dashboard on the passenger's side of the vehicle. In addition to fractures or dislocations of the knees, you should be MOST suspicious for: A. anterior hip dislocation. B. posterior hip dislocation. C. a thoracic spine fracture. D. fracture of the tibia or fibula.

B

A Colles fracture involves a fracture of the: A. distal ulna. B. distal radius. C. proximal radius. D. radius and ulna.

B

A _______ is a disruption of a joint in which the bone ends are no longer in contact. A. torn ligament B. dislocation C. fracture dislocation D. sprain

B

A ____________ is a musculoskeletal injury in which there is partial or temporary separation of the bone ends as well as partial stretching or tearing of the supporting ligaments. A. strain B. sprain C. fracture D. dislocation

B

A contusion to a patient's forehead along with a spider-webbed windshield suggests possible injury to the: A. nose B. brain C. face D. heart

B

A driver involved in a rollover motor vehicle crash will MOST likely experience serious injuries or death if he or she: A. experiences multiple impacts. B. is ejected or partially ejected. C. remains within the vehicle. D. is wearing only a lap belt.

B

A fall from more than _____ times the patient's height is considered to be significant. A. two B. three C. four D. five

B

A fracture caused by minimal force that is associated with diseases such as cancer and osteoporosis is called a __________ fracture. A. greenstick B. pathologic C. transverse D. comminuted

B

A fracture is MOST accurately defined as a(n): A. total loss of function in a bone. B. break in the continuity of the bone. C. disruption in the midshaft of a bone. D. abnormality in the structure of a bone.

B

A patient injured her knee while riding a bicycle. She is lying on the ground, has her left leg flexed, is in severe pain, and cannot move her leg. Your assessment reveals obvious deformity to her left knee. Distal pulses are present and strong. The MOST appropriate treatment for her injury involves: A. wrapping her entire knee area with a pillow. B. splinting the leg in the position in which it was found. C. straightening her leg and applying two rigid board splints. D. straightening her leg and applying and inflating an air splint.

B

A patient tripped, fell, and landed on her elbow. She is in severe pain and has obvious deformity to her elbow. You should: A. assess distal pulses. B. manually stabilize her injury. C. assess her elbow for crepitus. D. apply rigid board splints to her arm.

B

A person who experiences a calcaneus fracture after jumping and landing on his or her feet would MOST likely experience an accompanying fracture of the: A. thoracic spine. B. lumbar spine. C. coccygeal spine. D. symphysis pubis.

B

A supracondylar or intercondylar fracture is also known as a fracture of the: A. radial head. B. distal humerus. C. proximal radius. D. olecranon process.

B

A young boy was riding his bicycle down the street when he hit a parked car. What was the second collision? A. The bike hitting the car. B. The bike rider hitting his bike or the car. C. The bike rider's internal organs against the solid structures of the body. D. The bike rider striking the pavement.

B

A young male experienced severe blunt chest trauma when his passenger car struck another vehicle head-on. During your inspection of the interior of his vehicle, you would MOST likely find: A. deployed airbags. B. steering wheel deformity. C. starring of the windshield. D. a crushed instrument panel.

B

A(n) _______ is a fracture in which the bone is broken into two or more fragments. A. greenstick B. comminuted C. pathologic D. epiphyseal

B

A(n) _______ may cause a fracture or discoloration at a distant point. A. direct blow B. indirect force C. twisting force D. high-energy injury

B

By what mechanism is a person injured when he or she falls from a significant height? A. Potential energy is created as the person is falling; the potential energy is then converted into kinetic energy upon impact. B. Potential energy is converted to kinetic energy; the kinetic energy is then converted into the work of bringing the body to a stop. C. As the person falls, the amount of kinetic energy is converted into work; work is then converted to kinetic energy upon impact. D. Kinetic energy is converted to potential energy; the potential energy is then converted into the work of bringing the body to a stop.

B

Common signs and symptoms of a sprain include all of the following, EXCEPT: A. swelling. B. deformity. C. ecchymosis. D. point tenderness.

B

During your assessment of a 29-year-old female with significant deformity to her left elbow, you are unable to palpate a radial pulse. Your transport time to the hospital is approximately 40 minutes. You should: A. splint the elbow in the position of deformity and transport immediately. B. apply gentle manual traction in line with the limb and reassess for a pulse. C. carefully straighten the injured arm and secure it with padded board splints. D. make two or three attempts to restore distal circulation by manipulating the elbow.

B

During your rapid secondary assessment of a 19-year-old female with multiple trauma, you note bilateral humeral deformities and a deformity to the left midshaft femur. Her skin is diaphoretic and her pulse is rapid and weak. Your partner has appropriately managed her airway and is maintaining manual stabilization of her head. The MOST appropriate treatment for this patient includes: A. applying and inflating the pneumatic antishock garment (PASG) and transporting. B. immobilizing her to a backboard and rapidly transporting. C. applying a traction splint to immobilize her femur. D. carefully splinting each of her deformed extremities.

B

Factors that should be considered when assessing a patient who has fallen include all of the following, EXCEPT: A. the height of the fall. B. the speed of the fall. C. the surface struck. D. the primary impact point.

B

If a dislocated shoulder has spontaneously reduced before your arrival, the only way to confirm the injury is by noting: A. distal circulation. B. the patient history. C. bruising to the shoulder. D. the presence of deformity.

B

Kinetic energy is a calculation of: A. weight and size. B. weight and speed. C. mass and weight. D. speed and force.

B

Optimally, on-scene time for critically injured patients should be less than _____ minutes. A. 5 B. 10 C. 15 D. 20

B

Regardless of the extent and severity of the damage to the skin, you should treat any injury that breaks the skin as a possible: A. closed fracture. B. open fracture. C. nondisplaced fracture. D. displaced fracture.

B

The driver of a sport utility vehicle lost control and struck a utility pole head-on. The driver was killed instantly. The passenger, a young female, is conscious and alert and has several small abrasions and lacerations to her left forearm. Treatment for the passenger should include: A. transport to a community hospital. B. transport to a trauma center. C. a secondary assessment at the scene. D. a focused exam of her forearm.

B

The index of suspicion is MOST accurately defined as: A. a predictable pattern that leads to serious injuries. B. your concern for potentially serious underlying injuries. C. the detection of less obvious life-threatening injuries. D. the way in which traumatic injuries occur.

B

The knee is especially susceptible to _______ injuries, which occur when abnormal bending or twisting forces are applied to the joint. A. tendon B. ligament C. dislocation D. fracture-dislocation

B

The purpose of splinting a fracture is to: A. reduce the fracture if possible. B. prevent motion of bony fragments. C. reduce swelling in adjacent soft tissues. D. force the bony fragments back into anatomic alignment.

B

When assessing a patient who experienced a blast injury, it is important to remember that: A. secondary blast injuries are usually the least obvious. B. primary blast injuries are the most easily overlooked. C. solid organs usually rupture from the pressure wave. D. primary blast injuries are typically the most obvious.

B

When assessing a patient with a possible fracture of the leg, the EMT should: A. assess proximal circulation. B. compare it to the uninjured leg. C. carefully move it to elicit crepitus. D. ask the patient to move the injured leg.

B

When splinting a possible fracture of the foot, it is MOST important for the EMT to: A. use a pillow as a splint. B. leave the toes exposed. C. apply a pneumatic splint. D. observe for tissue swelling.

B

When treating an open extremity fracture, you should: A. apply a splint and then dress the wound. B. dress the wound before applying a splint. C. irrigate the wound before applying a dressing. D. allow the material that secures the splint to serve as the dressing.

B

Which of the following is considered a type of impact from a motor vehicle collision? A. Ejection B. Rollover C. Crush D. Peneration

B

Which of the following is not considered a type of impact associated with a motorcycle crash? A. Head-on B. Rotational C. Controlled D. Ejection

B

Which of the following types of muscle is under direct voluntary control of the brain? A. cardiac B. skeletal C. smooth D. autonomic

B

Your awareness and concern for potentially serious obvious and underlying injuries is referred to as the: A. mechanism of injury. B. index of suspicion. C. scene size-up. D. general impression.

B

_____ impacts are probably the number one cause of death associated with motor vehicle collisions. A. Frontal B. Lateral C. Rear-end D. Rollover

B

_______ are held together in a tough fibrous structure known as a capsule. A. Tendons B. Joints C. Ligaments D. Bones

B

Which of the following open soft-tissue injuries is limited to the superficial layer of the skin and results in the least amount of blood loss? Select one: A. Incision B. Abrasion C. Laceration D. Avulsion

B. Abrasion

Which of the following statements regarding secondary brain injury is correct? Select one: A. Because cerebral edema develops quickly, it is considered to be a primary brain injury. B. Hypoxia and hypotension are the two most common causes of secondary brain injury. C. Signs are often present immediately after an impact to the head. D. It results from direct brain trauma following an impact to the head.

B. Hypoxia and hypotension are the two most common causes of secondary brain injury.

Compression injury is most likely due to which of the following? Select one: A. Hollow-organ rupture B. Improperly placed lab belt C. Stabbing D. Ejection of unrestrained driver

B. Improperly placed lab belt

Which of the following breathing patterns is MOST indicative of increased intracranial pressure? Select one: A. Increased rate with a normal inspiratory time and a prolonged expiratory time B. Irregular rate, pattern, and volume of breathing with intermittent periods of apnea C. Increased rate and depth with the distinct odor of acetone on the patient's breath D. Slow, shallow, occasional gasps that progress to prolonged periods of apnea

B. Irregular rate, pattern, and volume of breathing with intermittent periods of apnea

Which of the following statements regarding abdominal eviscerations is correct? Select one: A. Most eviscerations occur to the left upper quadrant. B. The protruding organs should be kept warm and moist. C. The organs should be replaced carefully to avoid heat loss. D. Adherent material is preferred when covering an evisceration.

B. The protruding organs should be kept warm and moist.

If direct pressure fails to immediately stop severe bleeding from an extremity, you should apply: Select one: A. a splint and elevate the extremity. B. a tourniquet proximal to the injury. C. digital pressure to a proximal artery. D. additional sterile dressings.

B. a tourniquet proximal to the injury.

A 39-year-old male was struck in the head by a baseball during a game. He is confused and has slurred speech. He has a large hematoma in the center of his forehead and cannot remember the events preceding the injury. After manually stabilizing his head and assessing his airway, you should: Select one: A. palpate his radial pulses. B. administer high-flow oxygen. C. apply ice to the hematoma. D. perform a neurologic exam.

B. administer high-flow oxygen.

An open pneumothorax occurs when: Select one: A. extreme pleural pressure causes the lung to rupture. B. air enters the pleural space from outside the body. C. air enters the pleural space from a perforated lung. D. a fractured rib perforates the tissue of the lung.

B. air enters the pleural space from outside the body.

A young male sustained a gunshot wound to the abdomen during an altercation. As your partner is assessing and managing his airway, you should control the obvious bleeding and then: Select one: A. auscultate bowel sounds. B. assess for an exit wound. C. apply a cervical collar. D. obtain baseline vital signs.

B. assess for an exit wound.

Your documentation on a sexual assault victim should _______. Select one: A. describe the status of the suspect(s) B. be objective and factual C. include your opinion of the nature of the incident D. be subjective and summarize the crime

B. be objective and factual

The musculoskeletal system refers to the: Select one: A. involuntary muscles of the nervous system. B. bones and voluntary muscles of the body. C. nervous system's control over the muscles. D. connective tissue that supports the skeleton.

B. bones and voluntary muscles of the body.

When caring for a patient whose arm is covered with a dry chemical, you should: Select one: A. quickly irrigate the arm with large amounts of water. B. brush away the chemical before flushing with water. C. use forceful streams of water to remove the chemical. D. deactivate the chemical with a 5% vinegar solution.

B. brush away the chemical before flushing with water.

Your presence is requested by law enforcement to assess a 33-year-old female who was sexually assaulted. The patient is conscious and obviously upset. As you are talking to her, you note an impressive amount of blood on her clothes in the groin area. Her blood pressure is 98/58 mm Hg, her pulse is 130 beats/min, and her respirations are 24 breaths/min. You should: Select one: A. visualize the vaginal area and pack the vagina with sterile dressings. B. control any external bleeding, administer oxygen, and transport at once. C. allow her to change her clothes and take a shower before you transport. D. arrange for a rape crisis center representative to speak with the patient.

B. control any external bleeding, administer oxygen, and transport at once.

The MOST significant hazard associated with splinting is: Select one: A. compression of nerves, tissues, and vasculature. B. delaying transport of a critically injured patient. C. reduction in circulation distal to the injury site. D. aggravation of the injury or worsened pain.

B. delaying transport of a critically injured patient.

You are transporting a 42-year-old male who experienced blunt abdominal trauma. He is receiving oxygen at 12 L/min via a nonrebreathing mask and full spinal precautions have been applied. During your reassessment, you note his level of consciousness has decreased and his respirations have become shallow. You should: Select one: A. perform a comprehensive secondary assessment to determine why his clinical status has changed. B. insert an airway adjunct if he will tolerate it and begin assisting his ventilations with a BVM. C. suction his oropharynx to ensure it is clear of secretions and then increase the oxygen flow rate to 15 L/min. D. reassess his vital signs and then notify the receiving hospital of the change in his clinical status.

B. insert an airway adjunct if he will tolerate it and begin assisting his ventilations with a BVM.

Once a cervical collar has been applied to a patient with a possible spinal injury, it should not be removed unless: Select one: A. sensory and motor functions remain intact. B. it causes a problem managing the airway. C. lateral immobilization has been applied. D. the patient adamantly denies neck pain.

B. it causes a problem managing the airway.

You arrive at the scene of a major motor vehicle crash. The patient, a 50-year-old female, was removed from her vehicle prior to your arrival. Bystanders who removed her state that she was not wearing a seatbelt. The patient is unresponsive, tachycardic, and diaphoretic. Your assessment reveals bilaterally clear and equal breath sounds, a midline trachea, and collapsed jugular veins. You should be MOST suspicious that this patient has experienced a: Select one: A. pericardial tamponade. B. laceration of the aorta. C. massive hemothorax. D. tension pneumothorax.

B. laceration of the aorta.

In pediatric patients, the liver and spleen are _______. Select one: A. more protected by the thorax compared to adults B. larger in proportion to the abdomen C. less likely to bleed when injured D. smaller in proportion to the abdomen

B. larger in proportion to the abdomen

If a patient with a chest injury is only able to inhale small amounts of air per breath, he or she: Select one: A. often breathes at a slower rate because of lung damage caused by the injury. B. must increase his or her respiratory rate to maintain adequate minute volume. C. will eliminate more carbon dioxide than if he or she were breathing deeply. D. will maintain adequate minute volume if his or her respiratory rate stays the same.

B. must increase his or her respiratory rate to maintain adequate minute volume.

While en route to a major motor vehicle crash, an on-scene police officer advises you that a 6-year-old male who was riding in the front seat is involved. He further states that the child was only wearing a lap belt and that the air bag deployed. On the basis of this information, you should be MOST suspicious that the child has experienced: Select one: A. lower extremity fractures. B. neck and facial injuries. C. blunt trauma to the head. D. open abdominal trauma.

B. neck and facial injuries.

When assessing an elderly patient who fell, it is important to remember that: Select one: A. any fall in the elderly is considered to be high-energy trauma. B. osteoporosis can cause a fracture as a result of a fall from a standing position. C. bilateral hip fractures usually occur when an elderly person falls. D. elderly patients who fall usually have a secondary head injury.

B. osteoporosis can cause a fracture as a result of a fall from a standing position.

A spinal cord injury at the level of C7 would MOST likely result in: Select one: A. immediate cardiac arrest. B. paralysis of the intercostal muscles. C. paralysis of the diaphragm. D. paralysis of all the respiratory muscles.

B. paralysis of the intercostal muscles.

You have sealed the open chest wound of a 40-year-old male who was stabbed in the anterior chest. Your reassessment reveals that he is experiencing increasing respiratory distress and tachycardia, and is developing cyanosis. You should: Select one: A. begin ventilatory assistance. B. partially remove the dressing. C. call for a paramedic ambulance. D. begin rapid transport at once.

B. partially remove the dressing.

Burns to pediatric patients are generally considered more serious than burns to adults because: Select one: A. most burns in children are the result of child abuse. B. pediatric patients have more surface area relative to total body mass. C. pediatric patients have a proportionately larger volume of blood. D. pediatric patients are more prone to hyperthermia.

B. pediatric patients have more surface area relative to total body mass.

In order to avoid exacerbating a patient's injury, it is especially important to use extreme caution when providing positive-pressure ventilation to patients with a: Select one: A. cardiac tamponade. B. pneumothorax. C. myocardial contusion. D. flail chest.

B. pneumothorax.

A 28-year-old male was struck in the chest with a baseball bat during an altercation. He is conscious and alert and complains of severe chest pain. Your assessment reveals a large area of ecchymosis over the sternum and a rapid, irregular pulse. In addition to providing supplemental oxygen, you should: Select one: A. apply an AED and assess his BP. B. prepare for immediate transport. C. apply bulky dressings to the sternum. D. determine if he has cardiac problems.

B. prepare for immediate transport.

During your assessment of a patient who was shot in the abdomen, you notice a large entrance wound with multiple small puncture wounds surrounding it. This wound pattern is MOST consistent with a: Select one: A. .22-caliber pistol. B. shotgun. C. handgun. D. .357 magnum.

B. shotgun.

Significant trauma to the face should increase the EMT's index of suspicion for a(n): Select one: A. displaced mandible. B. spinal column injury. C. airway obstruction. D. basilar skull fracture.

B. spinal column injury.

All of the following are hollow abdominal organs, EXCEPT the: Select one: A. ureters. B. spleen. C. bladder. D. stomach.

B. spleen.

The Adam's apple is: Select one: A. the lower part of the larynx that is formed by the cricoid cartilage. B. the upper part of the larynx that is formed by the thyroid cartilage. C. below the thyroid cartilage and forms the upper part of the trachea. D. the small indentation in between the thyroid and cricoid cartilages.

B. the upper part of the larynx that is formed by the thyroid cartilage.

According to the Association of Air Medical Services, you should consider air medical transport of a trauma patient if: Select one: A. the patient requires advanced life support care and stabilization, and the nearest ALS-ground ambulance is more than 5 to 10 minutes away. B. traffic conditions hamper the ability to get the patient to a trauma center by ground within the ideal time frame for the best clinical outcome. C. ground transport will take your ambulance out of service for an extended period of time, regardless of the severity of the patient's injuries. D. he or she was involved in a motor vehicle crash in which another occupant in the same vehicle was killed, even if your patient's injuries are minor.

B. traffic conditions hamper the ability to get the patient to a trauma center by ground within the ideal time frame for the best clinical outcome.

If your patient swallows blood following facial trauma, there is an increased risk of ________. Select one: A. altered LOC B. vomiting C. hypotension D. GI trauma

B. vomiting

depth and extent

Burns are classified according to

pediatric patients have more surface area relative to total body mass

Burns to pediatric patients are generally considered more serious than burns to adults because:

"For every action, there is an equal and opposite reaction" is: A. Newton's first law. B. Newton's second law. C. Newton's third law. D. a false statement.

C

20-year-old man has major open facial injuries after his vehicle struck a tree head-on. Which of the following findings within the car would MOST likely explain his injury pattern? A. Deployed airbag B. Bent steering wheel C. Nonintact windshield D. Crushed instrument panel

C

A 15-year-old female was struck by a small car while riding her bicycle. She was wearing a helmet and was thrown to the ground, striking her head. In addition to managing problems associated with airway, breathing, and circulation, it is MOST important for you to: A. obtain baseline vital signs. B. inspect the helmet for cracks. C. stabilize her entire spine. D. leave her bicycle helmet on.

C

A 17-year-old football player collided with another player and has pain to his left clavicular area. He is holding his arm against his chest and refuses to move it. Your assessment reveals obvious deformity to the midshaft clavicle. After assessing distal pulse, sensory, and motor functions, you should: A. perform a rapid secondary assessment. B. straighten his arm and apply a board splint. C. immobilize the injury with a sling and swathe. D. place a pillow under his arm and apply a sling.

C

A 30-year-old male sustained a stab wound to the neck when he was attacked outside a nightclub. During your assessment, you should be MOST alert for: A. alterations in his mental status. B. damage to internal structures. C. potential airway compromise. D. injury to the cervical spine.

C

A 76-year-old male experienced sudden pain to his left thigh when he was standing in line at the grocery store. Your assessment reveals ecchymosis and deformity to the distal aspect of his left femur, just above the knee. Distal circulation and sensory and motor functions are intact. The MOST appropriate method of splinting this injury involves: A. applying a traction splint to realign the deformity. B. applying and fully inflating the pneumatic antishock garment (PASG). C. applying padded board splints to both sides of the leg. D. binding the legs together and elevating them 6″ to 8″.

C

A 77-year-old woman slipped and fell on a throw rug and landed on her left hip. She denies striking her head or losing consciousness. Assessment of her left leg reveals that it is shortened and externally rotated. Distal pulses, sensory, and motor functions are intact. You should: A. manually stabilize her left leg, apply a traction splint, and then secure her to a long backboard or scoop. B. carefully slide a long backboard underneath her, keep her in a supine position, and apply a splint to her leg. C. place her onto a scoop stretcher, pad around her left hip with pillows, and secure her to the scoop with straps. D. bind both of her legs together with triangular bandages and carefully secure her onto the ambulance stretcher.

C

A _______ is an injury to the ligaments, the articular capsule, the synovial membrane, and the tendons crossing the joint. A. dislocation B. strain C. sprain D. torn ligament

C

A motorcyclist crashed his bike and has closed deformities to both of his midshaft femurs. He is conscious, but restless; his skin is cool and clammy; and his radial pulses are rapid and weak. The MOST appropriate splinting technique for this patient involves: A. applying rigid board splints. B. applying two traction splints. C. securing him to a long backboard. D. immobilizing his femurs with air splints.

C

A small compact car was involved in a rollover crash. As you are approaching the vehicle, you note that the roof is significantly collapsed. The patient, a 29-year-old male, is complaining of severe pain in his neck and to the top of his head as well as numbness and tingling in his extremities. Witnesses who removed the patient from the vehicle state that he was wearing his seatbelt. What injury mechanism is MOST likely responsible for this patient's condition? A. Impact of the head against the steering wheel B. Whiplash injury to the neck during the rollover C. Compression of the head against the roof D. Lateral bending of the neck during the crash

C

A subluxation occurs when: A. ligaments are partially severed. B. a fracture and a dislocation exist. C. a joint is incompletely dislocated. D. a bone develops a hairline fracture.

C

A young male sustained a gunshot wound to the abdomen during an altercation with a rival gang member. As your partner is assessing and managing his airway, you should control the obvious bleeding and then: A. auscultate bowel sounds. B. apply a cervical collar. C. assess for an exit wound. D. obtain baseline vital signs.

C

A(n) _______ is also known as a hairline fracture. A. closed fracture. B. open fracture. C. nondisplaced fracture. D. displaced fracture.

C

A(n) __________ fracture occurs in the growth section of a child's bone and may lead to bone growth abnormalities. A. greenstick B. diaphyseal C. epiphyseal D. metaphyseal

C

Although substantial ligament damage always occurs with a knee dislocation, the more urgent injury is to the _______ artery, which is often lacerated or compressed by the displaced tibia. A. tibial B. femoral C. popliteal D. dorsalis pedis

C

Assessing a person's neurovascular status following a musculoskeletal injury includes all of the following, EXCEPT: A. assessing motor function. B. assessing sensory function. C. evaluating proximal pulses. D. determining capillary refill.

C

Atrophy is a condition that occurs when: A. increased use of skeletal muscle causes an increase in its strength. B. the tendons that attach muscle to bone become stretched or injured. C. muscle decreases in size and function because of disease or trauma. D. carbon dioxide, lactic acid, and other wastes accumulate in the muscle.

C

Crepitus and false motion are: A. indicators of a severe sprain. B. only seen with open fractures. C. positive indicators of a fracture. D. most common with dislocations.

C

During your rapid secondary assessment of a 30-year-old male who fell 25′, you note crepitus when palpating his pelvis. Your partner advises you that the patient's blood pressure is 80/50 mm Hg and his heart rate is 120 beats/min and weak. After completing your assessment, you should: A. defer spinal immobilization and transport to a trauma center. B. perform a focused physical exam with emphasis on the pelvis. C. stabilize the pelvis with a pelvic binder and protect the spine. D. log roll the patient onto a long backboard and transport at once

C

Energy can be: A. created. B. destroyed. C. converted. D. all of the above.

C

Fractures are almost always associated with _______ of the surrounding soft tissue. A. laceration B. crepitus C. ecchymosis D. swelling

C

In which of the following situations should the EMT splint an injured limb in the position of deformity? A. when distal circulation and neurological functions are absent B. if transport time to the hospital is greater than 20 to 30 minutes C. if resistance is encountered or the patient experiences severe pain D. if a traction splint will be used to immobilize the injured extremity

C

In-line _______ is the act of exerting a pulling force on a body structure in the direction of its normal alignment. A. stabilization B. immobilization C. traction D. direction

C

Internal injuries caused by gunshot wounds are difficult to predict because: A. the area of damage is usually smaller than the bullet. B. the caliber of the bullet is frequently unknown. C. the bullet may tumble or ricochet within the body. D. exit wounds caused by the bullet are usually small.

C

Joints are bathed and lubricated by _______ fluid. A. cartilaginous B. articular C. synovial D. cerebrospinal

C

Severe abrasion injuries can occur when motorcycle riders are slowed after a collision by road drag. Road drag is most often associated with which type of motorcycle impact? A. Head-on collision B. Angular collision C. Ejection D. Controlled crash

C

Medium-velocity penetrating injuries may be caused by a: A. knife. B. military assault rifle. C. handgun. D. sling-shot.

C

Passengers who are seated in the rear of a vehicle and are wearing only lap belts have a higher incidence of injuries to the: A. lumbar and coccygeal spine. B. thoracic and sacral spine. C. thoracic and lumbar spine. D. lumbar and sacral spine.

C

Patients suffering from an open wound to the neck may suffer from all of the following EXCEPT: A. significant bleeding. B. air embolism. C. tension pneumothorax. D. subcutaneous crepitation.

C

The _______ is one of the most commonly injured joints. A. knee B. elbow C. ankle D. hip

C

The energy of a moving object is called: A. potential energy. B. thermal energy. C. kinetic energy. D. work.

C

The three collisions in a frontal impact include all of the following EXCEPT: A. car striking object. B. passenger striking vehicle. C. air bag striking passenger. D. internal organs striking solid structures of the body.

C

There is often a significant amount of blood loss, as much as _______ mL, after a fracture of the shaft of the femur. A. 100 to 250 B. 250 to 500 C. 500 to 1,000 D. 100 to 1,500

C

Traction splints are used primarily to immobilize and secure fractures of the: A. hip. B. pelvis. C. femur. D. humerus.

C

Two of the MOST common mechanisms of injury for blunt trauma are: A. motor vehicle collisions and stabbings. B. low-caliber gunshot wounds and falls. C. falls and motor vehicle collisions. D. gunshot wounds and vehicle ejections.

C

When caring for patients who have fallen, you must identify the _______ and the mechanism of injury so that you will not overlook associated injuries. A. site of injury B. height of fall C. point of contact D. twisting forces

C

When evaluating the mechanism of injury of a car versus pedestrian collision, you should first: A. determine if the patient was struck and pulled under the vehicle. B. determine if the patient was propelled away from the vehicle. C. approximate the speed of the vehicle that struck the pedestrian. D. evaluate the vehicle that struck the patient for structural damage.

C

Which of the following injuries would MOST likely occur as a direct result of the third collision in a motor vehicle crash? A. Forehead lacerations B. Extremity fractures C. Aortic rupture D. Flail chest

C

When treating a patient who experienced a pulmonary blast injury, you should: A. suspect an accompanying cardiac tamponade. B. use a demand valve to ventilate the patient. C. avoid giving oxygen under positive pressure. D. administer large amounts of intravenous fluid.

C

Which of the following fractures has the greatest potential for internal blood loss and shock? A. hip B. femur C. pelvis D. humerus

C

Which of the following statements regarding low-energy penetrating injuries is correct? A. Exit wounds are typically easy to locate with low-energy penetrating injuries. B. It is usually easy to differentiate between an entrance wound and an exit wound. C. The area of injury is usually close to the path the object took through the body. D. Internal injuries caused by low-velocity bullets are usually easy to predict.

C

Which of the following statements regarding shoulder dislocations is MOST correct? A. Posterior dislocations are most common. B. They are caused by forced arm adduction. C. Most shoulder dislocations occur anteriorly. D. They involve the acromion process and humerus.

C

You are called to the local assisted living facility where a 94-year-old man has fallen. He is alert and oriented and denies passing out. His respirations are 18 breaths/min and regular. Pulse is 106 beats/min, regular, and strong. Distal pulses are present. He states that he was walking and heard a pop and fell to the floor. You suspect a(n): A. greenstick B. comminuted C. pathologic D. epiphyseal

C

_______ is the most reliable indicator of an underlying fracture. A. Crepitus B. Deformity C. Point tenderness D. Absence of distal pulse

C

You have applied a zippered air splint to a patient's left arm. During transport, the patient complains of increased numbness and tingling in his left hand. You reassess distal circulation and note that it remains present. Your MOST appropriate action should be to: A. elevate the injured arm and reassess distal sensory function. B. inflate the splint with more air until the patient is comfortable. C. assess the amount of air in the splint and let air out as necessary. D. remove the air splint and reimmobilize with padded board splints.

C

You respond to a soccer game for a 16-year-old male with severe ankle pain. When you deliver him to the hospital, the physician tells you that he suspects a sprain. This means that: A. there is a disruption of the joint and the bone ends are no longer in contact. B. the patient has an incomplete fracture that passes only partway through the bone. C. stretching or tearing of the ligaments with partial or temporary dislocation of the bone ends has occurred. D. the muscles of the ankle have been severely stretched, resulting in displacement of the bones from the joint.

C

Your patient has a Glasgow Coma Scale (GCS) score of 13, a systolic blood pressure of 80 mm Hg, and a respiratory rate of 8 breaths/min, his Revised Trauma Score (RTS) is: A. 8. B. 11. C. 9. D. 10.

C

Your quick primary assessment of the patient and evaluation of the _____ can help to direct lifesaving care and provide critical information to the hospital staff. A. environment B. index of suspicion C. mechanism of injury D. abdominal area

C

A 21-year-old male was working in an auto repair shop and sustained radiator burns to the anterior aspect of both arms and to his anterior chest. According to the rule of nines, this patient has burns that cover _____ of his BSA. Select one: A. 27% B. 45% C. 18% D. 36%

C. 18%

Significant vital sign changes will occur if the typical adult acutely loses more than ______ of his or her total blood volume. Select one: A. 10% B. 5% C. 20% D. 15%

C. 20%

Your patient has a Glasgow Coma Scale (GCS) score of 13, a systolic blood pressure of 80 mm Hg, and a respiratory rate of 8 breaths/min. His Revised Trauma Score (RTS) is: Select one: A. 10. B. 11. C. 9. D. 8.

C. 9.

In which of the following patients should you remove an impaled object? Select one: A. An apneic patient with a shard of glass impaled in the abdomen B. A conscious and alert patient with a fishhook impaled in the eye C. A pulseless and apneic patient with a knife impaled in the back D. A semiconscious patient with an ice pick impaled in the chest

C. A pulseless and apneic patient with a knife impaled in the back

When performing a full body scan, you should assess for ________. Select one: A. SAMPLE B. OPQRST C. DCAP-BTLS D. AVPU

C. DCAP-BTLS

Following a stab wound to the left anterior chest, a 25-year-old male presents with a decreased level of consciousness and signs of shock. Which of the following additional assessment findings should increase your index of suspicion for a cardiac tamponade? Select one: A. A rapid, irregular pulse B. Widening pulse pressure C. Engorged jugular veins D. Diminished breath sounds

C. Engorged jugular veins

Which organ or organ system has the greatest tolerance for lack of perfusion (shock)? Select one: A. Brain B. Kidneys C. Gastrointestinal system D. Skeletal muscle

C. Gastrointestinal system

In which of the following situations should the EMT splint an injured limb in the position of deformity? Select one: A. When distal circulation and neurological functions are absent B. If transport time to the hospital is greater than 20 to 30 minutes C. If resistance is encountered or the patient experiences severe pain D. If a traction splint will be used to immobilize the injured extremity

C. If resistance is encountered or the patient experiences severe pain

Which of the following organs would MOST likely bleed profusely when injured? Select one: A. Bladder B. Stomach C. Liver D. Intestine

C. Liver

When should you visually inspect the external genitalia on your patient? Select one: A. Only when ordered by medical direction B. Anytime the patient agrees to treatment and transport C. Only when there is a complaint of severe pain or other injury D. Always during the secondary assessment

C. Only when there is a complaint of severe pain or other injury

Which of the following musculoskeletal injuries would pose the greatest threat to a patient's life? Select one: A. Nondisplaced long bone fractures B. An amputated extremity C. Pelvic fracture with hypotension D. Bilateral femur fractures

C. Pelvic fracture with hypotension

Which of the following interventions is the MOST critical to the outcome of a patient with multisystem trauma? Select one: A. Intravenous fluid administration B. Early administration of oxygen C. Rapid transport to a trauma center D. Elevation of the lower extremities

C. Rapid transport to a trauma center

Which section of the heart receives deoxygenated blood? Select one: A. Atria B. Left C. Right D. Ventricles

C. Right

Which types of motor vehicle collisions present the greatest potential for multiple impacts? Select one: A. Lateral and rollover B. Frontal and rotational C. Rotational and rollover D. Rear-end and rotational

C. Rotational and rollover

Which of the following processes occurs during the inflammation phase of the healing process? Select one: A. The veins and arteries at the injury site constrict and platelets aggregate, which stops bleeding and causes a temporary increase in the size of the wound. B. The blood vessels in and around the injury site constrict, which forces bacteria and other microorganisms away, thus preventing significant infection. C. The immune system releases histamines, which cause vasodilation and increased capillary permeability, resulting in local redness and swelling. D. White blood cells are forced away from the injury site, thus allowing an increase in the flow of red blood cells, resulting in increased blood flow.

C. The immune system releases histamines, which cause vasodilation and increased capillary permeability, resulting in local redness and swelling

Which of the following statements regarding gunshot wounds is correct? Select one: A. The size of a bullet has the greatest impact on the injury produced. B. High-velocity bullets will cause less severe internal injuries. C. The speed of a bullet has the greatest impact on the injury produced. D. Low-velocity bullets will cause the greatest amount of trauma.

C. The speed of a bullet has the greatest impact on the injury produced.

Which of the following statements regarding anterior nosebleeds is correct? Select one: A. They are usually caused by a fracture of the basilar skull. B. They are usually severe and require aggressive treatment to control. C. They usually originate from the septum area and bleed slowly. D. They cause blood to drain into the posterior pharynx.

C. They usually originate from the septum area and bleed slowly.

An indicator of an expanding intracranial hematoma or rapidly progressing brain swelling is: Select one: A. acute unilateral paralysis following the injury. B. a progressively lowering blood pressure. C. a rapid deterioration of neurologic signs. D. an acute increase in the patient's pulse rate.

C. a rapid deterioration of neurologic signs.

Common signs and symptoms of a serious head injury include all of the following, EXCEPT: Select one: A. CSF leakage from the ears. B. decerebrate posturing. C. a rapid, thready pulse. D. widening pulse pressure.

C. a rapid, thready pulse.

The MOST significant complication associated with facial injuries is: Select one: A. damage to the eyes. B. cervical spine injury. C. airway compromise. D. mandibular immobility.

C. airway compromise.

With regard to musculoskeletal injuries, the zone of injury is defined as the: Select one: A. part of the body that sustained secondary injury. B. area of obvious deformity over the site of impact. C. area of soft-tissue damage surrounding the injury. D. exact part of the bone or joint that was disrupted.

C. area of soft-tissue damage surrounding the injury.

A man jumped from the roof of his house and landed on his feet. He complains of pain to his heels, knees, and lower back. This mechanism of injury is an example of: Select one: A. distraction. B. hyperflexion. C. axial loading. D. hyperextension.

C. axial loading.

A 30-year-old female presents with redness, inflammation, and pain to her left eye. During your assessment, you note that she is having difficulty keeping her eyes open. You should suspect that she is experiencing: Select one: A. acute retinitis. B. a detached retina. C. conjunctivitis. D. a corneal abrasion.

C. conjunctivitis.

The thoracic cavity is separated from the abdominal cavity by the: Select one: A. intercostal margin. B. costovertebral angle. C. diaphragm. D. anterior rib cage.

C. diaphragm.

A 52-year-old unrestrained female struck the steering wheel with her face when her truck collided with another vehicle. She has obvious swelling to her face and several dislodged teeth. A visual exam of her mouth reveals minimal bleeding. She is conscious and alert with a blood pressure of 130/80 mm Hg, a pulse of 110 beats/min, and respirations of 22 breaths/min with adequate tidal volume. You should: Select one: A. assist ventilations with a BVM device, immobilize her spine, suction her oropharynx for 30 seconds, and transport. B. apply oxygen via a nonrebreathing mask, suction her airway as needed, disregard the dislodged teeth, and transport. C. fully immobilize her spine, attempt to locate the dislodged teeth, suction as needed, and transport. D. fully immobilize her spine, irrigate her empty tooth sockets, attempt to locate the dislodged teeth, and transport.

C. fully immobilize her spine, attempt to locate the dislodged teeth, suction as needed, and transport.

A 67-year-old male presents with weakness, dizziness, and melena that began approximately 2 days ago. He denies a history of trauma. His blood pressure is 90/50 mm Hg and his pulse is 120 beats/min and thready. You should be MOST suspicious that this patient is experiencing: Select one: A. an aortic aneurysm. B. acute appendicitis. C. gastrointestinal bleeding. D. intrathoracic hemorrhaging.

C. gastrointestinal bleeding.

You are assessing a 59-year-old male and note that his pupils are unequal. He is conscious and alert. When obtaining his medical history, it is MOST pertinent to ask him if he: Select one: A. noticed the change during a meal. B. regularly sees a family physician. C. has a history of eye surgeries. D. is allergic to any medications.

C. has a history of eye surgeries.

You respond to a residence for a 40-year-old female who was assaulted by her husband; the scene has been secured by law enforcement. Upon your arrival, you find the patient lying supine on the floor in the kitchen. She is semiconscious with severely labored breathing. Further assessment reveals a large bruise to the left anterior chest, jugular venous distention, and unilaterally absent breath sounds. As your partner is supporting her ventilations, you should: Select one: A. perform a focused secondary exam. B. insert an oropharyngeal airway. C. immediately request ALS support. D. obtain a set of baseline vital signs.

C. immediately request ALS support.

Abdominal pain, vomiting, and fever are most likely due to _______. Select one: A. hemorrhage B. hypovolemia C. infection D. evisceration

C. infection

Following a blunt injury to the head, a 22-year-old female is confused and complains of a severe headache and nausea. On the basis of these signs and symptoms, you should be MOST concerned with the possibility of: Select one: A. a fracture of the skull. B. airway compromise. C. intracranial bleeding. D. spinal cord injury.

C. intracranial bleeding.

A factory worker was splashed in the eyes with a strong acid chemical. He complains of intense pain and blurred vision. Your ambulance does not carry bottles of sterile saline or water. You should: Select one: A. neutralize the acid chemical in his eye with an alkaline chemical. B. mix baking soda with water and irrigate his eyes with the solution. C. irrigate both eyes continuously for 20 minutes with plain water. D. flush both eyes with an alcohol-based solution and transport.

C. irrigate both eyes continuously for 20 minutes with plain water.

A simple pneumothorax: Select one: A. heals on its own without any treatment. B. often has a nontraumatic cause. C. is commonly caused by blunt chest trauma. D. is caused by penetrating chest trauma.

C. is commonly caused by blunt chest trauma.

A tight-fitting motorcycle helmet should be left in place unless: Select one: A. the patient complains of severe neck or back pain. B. the helmet is equipped with a full face shield or visor. C. it interferes with your assessment of the airway. D. the patient must be placed onto a long backboard.

C. it interferes with your assessment of the airway.

Placing a pregnant patient in a supine position during the third trimester of pregnancy: Select one: A. results in spontaneous urinary incontinence if the bladder is full. B. often causes hypotension secondary to cardiac compression. C. may decrease the amount of blood that returns to the heart. D. is recommended if the patient has severe abdominal discomfort.

C. may decrease the amount of blood that returns to the heart.

As you approach a young male who was involved in an industrial accident, you note that his eyes are closed and that he is not moving. You can see several large contusions to his arms, a laceration to his forehead with minimal bleeding, and a closed deformity to his right leg. You should: Select one: A. assess his pulse for rate, regularity, and quality. B. perform an immediate head-to-toe assessment. C. open his airway and assess his breathing status. D. apply high-flow oxygen and assess his injuries.

C. open his airway and assess his breathing status.

A 43-year-old man is experiencing a severe nosebleed. His blood pressure is 190/110 mm Hg and his heart rate is 90 beats/min and bounding. Preferred treatment for this patient includes: Select one: A. packing both nostrils with gauze pads until the bleeding stops. B. placing a rolled 4² × 4² dressing between his lower lip and gum. C. pinching the patient's nostrils and having him lean forward. D. having the patient pinch his own nostrils and then lie supine.

C. pinching the patient's nostrils and having him lean forward.

When assessing a patient who experienced a blast injury, it is important to remember that: Select one: A. solid organs usually rupture from the pressure wave. B. secondary blast injuries are usually the least obvious. C. primary blast injuries are the most easily overlooked. D. primary blast injuries are typically the most obvious.

C. primary blast injuries are the most easily overlooked.

A "hip" fracture is actually a fracture of the: Select one: A. pubic symphysis. B. femoral shaft. C. proximal femur. D. pelvic girdle.

C. proximal femur.

A man called EMS 12 hours after injuring his chest. Your assessment reveals a flail segment to the right side of the chest. The patient is experiencing respiratory distress and his oxygen saturation is 78%. His breath sounds are equal bilaterally and his jugular veins are normal. You should suspect: Select one: A. massive hemothorax. B. tension pneumothorax. C. pulmonary contusion. D. traumatic asphyxia.

C. pulmonary contusion.

A 6-year-old female was riding her bicycle and struck a clothesline with her throat. She is breathing, but with obvious difficulty. Your assessment reveals a crackling sensation in the soft tissues of her neck and facial cyanosis. In addition to the appropriate airway management, the intervention that will MOST likely improve her chance of survival is: Select one: A. requesting a paramedic ambulance. B. careful monitoring her vital signs. C. rapidly transporting her to the hospital. D. quickly immobilizing her spinal column.

C. rapidly transporting her to the hospital.

A 56-year-old male has an incomplete avulsion to his right forearm. After controlling any bleeding from the wound, you should: Select one: A. thoroughly irrigate the wound with sterile water and cover it with a sterile dressing. B. carefully remove the avulsed flap and wrap it in a moist, sterile trauma dressing. C. replace the avulsed flap to its original position and cover it with a sterile dressing. D. carefully probe the wound to determine if the bleeding is venous or arterial.

C. replace the avulsed flap to its original position and cover it with a sterile dressing.

An 8-year-old male was bitten by a stray dog. He has a large laceration to the back of his left hand, which your partner covered with a sterile dressing and bandage. In addition to transporting the child to the hospital, you should: Select one: A. ask the child's father to try to locate the dog. B. advise the child that he will need rabies shots. C. report the incident to the appropriate authorities. D. administer oxygen via a nonrebreathing mask.

C. report the incident to the appropriate authorities

A 15-year-old female was struck by a small car while riding her bicycle. She was wearing a helmet and was thrown to the ground, striking her head. In addition to managing problems associated with airway, breathing, and circulation, it is MOST important for you to: Select one: A. leave her bicycle helmet on. B. obtain baseline vital signs. C. stabilize her entire spine. D. inspect the helmet for cracks.

C. stabilize her entire spine.

During your primary assessment of a 19-year-old unconscious male who experienced severe head trauma, you note that his respirations are rapid, irregular, and shallow. He has bloody secretions draining from his mouth and nose. You should: Select one: A. assist his ventilations with a BVM. B. pack his nostrils to stop the drainage of blood. C. suction his oropharynx for up to 15 seconds. D. immobilize his spine and transport immediately.

C. suction his oropharynx for up to 15 seconds.

Patients with chest injuries will often present with _______. Select one: A. Kussmaul respirations B. agonal respirations C. tachypnea D. Cheyne-Stokes respirations

C. tachypnea

Skeletal muscle is attached to the bone by tough, ropelike, fibrous structures called: Select one: A. ligaments. B. fascia. C. tendons. D. cartilage.

C. tendons.

blunt trauma

Closed chest injuries are typically caused by _______.

Hypovolemic shock occurs when: Select one: A. the patient's systolic blood pressure is less than 100 mm Hg. B. the clotting ability of the blood is enhanced. C. the body cannot compensate for rapid blood loss. D. at least 10% of the patient's blood volume is lost.

C. the body cannot compensate for rapid blood loss.

When transporting a patient with a facial injury, it is MOST important to be as descriptive as possible with the hospital regarding the patient's injuries because: Select one: A. most patients with facial trauma will need surgery. B. it saves time on repeat assessments at the hospital. C. they may need to call a specialist to see the patient. D. they must make arrangements for an ICU bed.

C. they may need to call a specialist to see the patient.

their intercostal muscles are not developed

Children are often "belly breathers" because _______.

a poorly placed lap belt

Compression injuries to the abdomen that occur during a motor vehicle crash are typically the result of:

A 22-year-old female was ejected from her car after striking a tree head-on. As you approach her, you note obvious closed deformities to both of her femurs. She is not moving and does not appear to be conscious. You should: A. apply manual stabilization to both of her femurs. B. administer oxygen and perform a rapid assessment. C. assess for a carotid pulse and assist her ventilations. D. stabilize her head and perform a primary assessment.

D

A 40-year-old man was standing near a building when it exploded. He has multiple injuries, including a depressed skull fracture, severe burns, and an impaled object in his abdomen. His head injury MOST likely occurred during the ___________ phase of the explosion. A. blast B. primary C. secondary D. tertiary

D

A 54-year-old male accidentally shot himself in the leg while cleaning his gun. Your assessment reveals a small entrance wound to the medial aspect of his right leg. The exit wound is on the opposite side of the leg and is actively bleeding. The patient complains of numbness and tingling in his right foot. You should: A. assess distal pulses as well as sensory and motor functions. B. manually stabilize the leg above and below the site of injury. C. gently manipulate the injured leg until the numbness dissipates. D. control the bleeding and cover the wound with a sterile dressing.

D

A patient complaining of chest tightness, coughing up blood, and subcutaneous emphysema following an explosion may be suffering from a: A. myocardial blast injury. B. ruptured tympanic membrane. C. ruptured peritoneal cavity. D. pulmonary blast injury.

D

A young male has a musculoskeletal injury and is unresponsive. You will NOT be able to assess: A. false motion. B. distal pulses. C. capillary refill. D. sensory and motor functions.

D

A(n) _______ fracture occurs in a growth section of a child's bone, which may prematurely stop growth if not properly treated. A. greenstick B. comminuted C. pathologic D. epiphyseal

D

A(n) _______ fracture produces actual deformity, or distortion, of the limb by shortening, rotating, or angulating it. A. closed fracture. B. open fracture. C. nondisplaced fracture. D. displaced fracture.

D

According to the American College of Surgeons, an adult trauma patient meets Level I criteria if he or she: A. has a systolic blood pressure of less than 110 mm Hg or a heart rate greater than 110 beats/min. B. was involved in a motor vehicle crash in which another patient in the same vehicle was killed. C. has a bleeding disorder or takes anticoagulant medications and has any blunt or penetrating injury. D. has a GCS score of less than or equal to 8 with a mechanism attributed to trauma.

D

All of the following are considered types of motorcycle impacts EXCEPT: A. head-on collision. B. angular collision. C. controlled crash. D. rear collision.

D

Always check neurovascular function: A. after any manipulation of the limb. B. before applying a splint. C. after applying a splint. D. all of the above.

D

An open fracture is MOST accurately defined as a fracture in which: A. bone ends protrude through the skin. B. a large laceration overlies the fracture. C. a bullet shatters the underlying bone. D. the overlying skin is no longer intact.

D

Approximately 25% of severe injuries to the aorta occur during: A. rear-end collisions. B. rollover collisions. C. frontal collisions. D. lateral collisions.

D

Bones are connected to other bones by bands of tough fibrous tissues called: A. bursa. B. tendons. C. cartilage. D. ligaments.

D

Burns from hot gases and respiratory injuries from inhaling toxic gas are associated with which type of blast injury? A. Primary B. Secondary C. Tertiary D. Miscellaneous

D

Death from a rollover motor vehicle crash is MOST often secondary to: A. crushing injuries. B. airbag-related trauma. C. multiple collisions to the interior of the car. D. ejection of the patient from the motor vehicle.

D

During your assessment of a patient who experienced a blast injury, you note that he has a depressed area to the front of his skull. This injury MOST likely occurred: A. during the secondary phase. B. during the primary phase. C. as a direct result of the pressure wave. D. during the tertiary phase.

D

Evaluation of the interior of a crashed motor vehicle during extrication will allow the EMT to: A. determine the vehicle's speed at the time of impact. B. recognize if the driver hit the brakes before impact. C. assess the severity of the third collision of the crash. D. identify contact points and predict potential injuries.

D

Following direct trauma to the left upper back, a 44-year-old male presents with diaphoresis and restlessness. His blood pressure is 100/50 mm Hg, his pulse rate is 120 beats/min and weak, and his respirations are 24 breaths/min and labored. Your assessment reveals abrasions and contusions over the left scapula. You should: A. hyperventilate him with a bag-mask device and monitor his oxygen saturation. B. apply high-flow oxygen, perform a detailed secondary assessment, and transport. C. focus your assessment on his scapulae and clavicles and apply high-flow oxygen. D. apply high-flow oxygen, consider spinal precautions, and transport without delay.

D

Force acting over a distance defines the concept of: A. potential energy. B. latent energy. C. kinetic energy. D. work.

D

If one or more occupants in the same vehicle are killed in a crash, the EMT should: A. rapidly assess only the survivors who have experienced obvious trauma. B. transport the survivors only if they have injuries or complain of pain. C. allow the survivors to refuse transport if they have no obvious injuries. D. suspect that all living occupants experienced the same serious trauma.

D

In contrast to a Level III trauma center, a Level I trauma center must: A. be able to stabilize patients before transferring to a higher level facility. B. be involved in trauma prevention programs. C. have access to an emergency physician within 30 minutes. D. have general surgeons that are in-house 24 hours a day.

D

The MOST reliable indicator of an underlying fracture is: A. guarding. B. severe swelling. C. obvious bruising. D. point tenderness.

D

The MOST significant hazard associated with splinting is: A. aggravation of the injury or worsened pain. B. reduction in circulation distal to the injury site. C. compression of nerves, tissues, and vasculature. D. delaying transport of a critically injured patient.

D

The disruption of a joint in which the bone ends are no longer in contact is called a: A. strain. B. sprain. C. fracture. D. dislocation.

D

What type of muscle contracts and relaxes to control the movement of the contents within its structures? A. cardiac B. skeletal C. striated D. smooth

D

What types of motor vehicle collisions present the greatest potential for multiple impacts? A. Rear-end and rotational B. lateral and rollover C. frontal and rotational D. rotational and rollover

D

When a driver is in a car equipped with an air bag, but is not wearing a seatbelt, he or she will MOST likely strike the __________ when the air bag deploys upon impact. A. windshield B. dashboard C. steering wheel D. door

D

When assessing a stab wound, it is important for the EMT to remember that: A. stabbings to an extremity are rarely associated with an exit wound. B. the majority of the internal trauma will be near the path of the knife. C. most stabbings are unintentional and cause less severe internal injury. D. more internal damage may be present than the external wound suggests.

D

When assessing the interior of a crashed motor vehicle for damage, you are gathering information regarding the: A. index of suspicion. B. kinetic energy. C. potential energy. D. mechanism of injury.

D

Which of the following is NOT considered appropriate use of air medical services? A. The distance to a trauma center is greater than 25 miles. B. Traffic/road conditions make it unlikely to get the patient to the hospital in a timely manner. C. There is a mass-casualty incident. D. The closest trauma center is 10 minutes away by ground transport.

D

Which of the following is a basic type of splint? A. Rigid B. Formable C. Traction D. All of the above

D

Which of the following is not considered one of the "6 Ps" of the musculoskeletal assessment? A. Pain B. Pulselessness C. Pressure D. Peristalsis

D

Which of the following is the MOST common cause of death from a blast injury? A. amputation B. burns C. chest trauma D. head trauma

D

Which of the following musculoskeletal injuries would MOST likely result in deformity? A. severe strain B. moderate sprain C. hairline fracture D. displaced fracture

D

Which of the following musculoskeletal injuries would pose the greatest threat to a patient's life? A. an amputated extremity B. bilateral femur fractures C. nondisplaced long bone fractures D. pelvic fracture with hypotension

D

Which of the following statements regarding gunshot wounds is correct? A. The size of a bullet has the greatest impact on producing injury. B. High-velocity bullets will cause less severe internal injuries. C. Low-velocity bullets will cause the greatest amount of trauma. D. The speed of a bullet has the greatest impact on producing injury.

D

Which of the following statements regarding striated muscle is correct? A. Striated muscle is involuntary because you have no conscious control over it. B. Most of the body's striated muscle is found within the walls of the blood vessels. C. Striated muscle tissue is attached directly to the bones by tough bands of cartilage. D. It forms the major muscle mass of the body and usually crosses at least one joint.

D

_______ produce severe damage to the skeleton, surrounding soft tissues, and vital internal organs. A. Direct blow B. Indirect force C. Twisting force D. High-energy injury

D

If applying a dressing to control the bleeding of a patient's arm, the EMT should ________. Select one: A. apply direct pressure first B. use large or small gauze pads or dressings depending upon the size of the wound C. cover the entire wound, above and below, with the dressing D. All of these answers are correct.

D. All of these answers are correct.

What mechanism(s) does the body use to control bleeding? Select one: A. Clotting B. Coagulation C. Vasoconstriction D. All of these answers are correct.

D. All of these answers are correct.

Which of the following is most likely to cause immediate death? Select one: A. Pulmonary contusion B. Aortic dissection C. Myocardial contusion D. Aortic rupture

D. Aortic rupture

Which of the following statements regarding crush syndrome is correct? Select one: A. With crush syndrome, massive blood vessel damage occurs following severe soft-tissue injuries, such as amputation of an extremity. B. Tissue damage that occurs in crush syndrome is severe, but kidney injury is unlikely because toxins are quickly eliminated from the body. C. Provided that a patient with a crush injury is freed from entrapment within 6 hours, the amount of tissue damaged is generally minimal. D. Compromised arterial blood flow leads to crush syndrome and can occur when an area of the body is trapped for longer than 4 hours.

D. Compromised arterial blood flow leads to crush syndrome and can occur when an area of the body is trapped for longer than 4 hours.

A 33-year-old male sustained an abdominal evisceration to the left lower quadrant of his abdomen after he was cut with a large knife. After appropriately managing his ABCs and assessing him for other life-threatening injuries, how you should care for his wound? Select one: A. Irrigate it with sterile water and cover it with a dry dressing. B. Carefully replace the exposed bowel into the abdomen and transport. C. Cover the exposed bowel and keep his legs in a straight position. D. Cover it with moist, sterile gauze and secure with an occlusive dressing.

D. Cover it with moist, sterile gauze and secure with an occlusive dressing.

Which of the following findings would be LEAST suggestive of the presence of high-energy trauma? Select one: A. Intrusion into the vehicle B. Dismounted seats C. Steering wheel deformity D. Deployment of the air bag

D. Deployment of the air bag

A 12-year-old male jumped approximately 12 feet from a tree and landed on his feet. He complains of pain to his lower back. What injury mechanism is MOST likely responsible for his back pain? Select one: A. Lateral impact to the spine B. Secondary fall after the initial impact C. Direct trauma to the spinal column D. Energy transmission to the spine

D. Energy transmission to the spine

Which of the following is a severe burn in a 65-year-old patient? Select one: A. Superficial burn to 30% of the BSA B. Full-thickness burn to 1% of the BSA C. Second-degree burn covering 10% of the BSA D. Partial-thickness burn to 20% of the BSA

D. Partial-thickness burn to 20% of the BSA

Which of the following statements regarding hemophilia is correct? Select one: A. Hemophilia is defined as a total lack of platelets. B. Hemophiliacs take aspirin to enhance blood clotting. C. Approximately 25% of the population has hemophilia. D. Patients with hemophilia may bleed spontaneously.

D. Patients with hemophilia may bleed spontaneously.

You should suspect a kidney injury anytime the patient presents with _______. Select one: A. abdominal distention B. nausea C. dyspnea D. a hematoma in the flank region

D. a hematoma in the flank region

A 54-year-old male experienced an avulsion to his penis when his foreskin got caught in the zipper of his pants. He was able to unzip his pants and remove the foreskin prior to your arrival. Your assessment reveals that he is in severe pain and that the avulsion is bleeding moderately. The MOST appropriate treatment for this patient includes: Select one: A. covering the avulsion with moist, sterile dressings. B. administering 100% oxygen via a nonrebreathing mask. C. requesting a paramedic to administer pain medication. D. applying direct pressure with a dry, sterile dressing.

D. applying direct pressure with a dry, sterile dressing.

During your assessment of a 29-year-old female with significant deformity to her left elbow, you are unable to palpate a radial pulse. Your transport time to the hospital is approximately 40 minutes. You should: Select one: A. carefully straighten the injured arm and secure it with padded board splints. B. make two or three attempts to restore distal circulation by manipulating the elbow. C. splint the elbow in the position of deformity and transport immediately. D. apply gentle manual traction in line with the limb and reassess for a pulse.

D. apply gentle manual traction in line with the limb and reassess for a pulse.

You have applied a dressing and roller-gauze bandage to a laceration on the arm of a young female. During transport, she begins to complain of numbness and tingling in her hand. You should: Select one: A. remove the gauze bandage and replace it with an elastic one. B. carefully manipulate her arm until the symptoms subside. C. remove the bandage and dressing and apply another one. D. assess distal circulation and readjust the bandage as needed.

D. assess distal circulation and readjust the bandage as needed.

A young male was shot in the abdomen by an unknown type of gun. He is semiconscious, has shallow breathing, and is bleeding externally from the wound. As you control the external bleeding, your partner should: Select one: A. perform a secondary assessment. B. apply a nonrebreathing mask. C. obtain baseline vital signs. D. assist the patient's ventilations.

D. assist the patient's ventilations.

The five sections of the spinal column, in descending order, are the: Select one: A. thoracic, cervical, lumbar, coccygeal, and sacral. B. cervical, coccygeal, thoracic, sacral, and lumbar. C. coccygeal, sacral, lumbar, thoracic, and cervical. D. cervical, thoracic, lumbar, sacral, and coccygeal.

D. cervical, thoracic, lumbar, sacral, and coccygeal.

You are transporting a patient with blunt abdominal trauma. The patient is unstable and is experiencing obvious signs and symptoms of shock. Your estimated time of arrival at the hospital is less than 10 minutes. After treating the patient appropriately, you should: Select one: A. begin documenting the call on the patient care form. B. perform a comprehensive secondary assessment. C. forgo the hospital radio report because of his condition. D. closely monitor him and reassess him frequently.

D. closely monitor him and reassess him frequently.

Difficulty breathing and a sunken appearance of the anterior abdominal wall is MOST indicative of a ruptured: Select one: A. aorta. B. spleen. C. stomach. D. diaphragm.

D. diaphragm.

A supracondylar or intercondylar fracture is also known as a fracture of the: Select one: A. olecranon process. B. radial head. C. proximal radius. D. distal humerus.

D. distal humerus.

According to the American College of Surgeons Committee on Trauma (ACS-COT), an adult trauma patient should be transported to the highest level of trauma center if he or she: Select one: A. was involved in a motor vehicle crash in which another patient in the same vehicle was killed. B. has a systolic blood pressure of less than 110 mm Hg or a heart rate greater than 110 beats/min. C. has a bleeding disorder or takes anticoagulant medications and has any blunt or penetrating injury. D. has a GCS score of less than or equal to 13 with a mechanism attributed to trauma.

D. has a GCS score of less than or equal to 13 with a mechanism attributed to trauma.

Approximately 25% of severe injuries to the aorta occur during: Select one: A. frontal collisions. B. rollover collisions. C. rear-end collisions. D. lateral collisions.

D. lateral collisions.

The upper jawbones are called the: Select one: A. mastoid. B. mandible. C. zygoma. D. maxillae.

D. maxillae.

You should be MOST suspicious that a patient has experienced a significant head injury if his or her pulse is: Select one: A. irregular. B. weak. C. rapid. D. slow.

D. slow.

Internal bleeding into a fractured extremity is MOST often controlled by: Select one: A. applying chemical ice pack. B. keeping the patient warm. C. applying a tourniquet. D. splinting the extremity.

D. splinting the extremity.

A burn that is characterized by redness and pain is classified as a: Select one: A. second-degree burn. B. partial-thickness burn. C. full-thickness burn. D. superficial burn.

D. superficial burn.

Beck's triad

Distended jugular veins, narrowing pulse pressure, and muffled heart tones are collectively known as _______.

administer oxygen and prepare for rapid transport

During your assessment of a 22-year-old male who was assaulted, you note widespread contusions and abrasions to his face, chest, and abdomen. His pulse is rapid and weak, and his skin is cool and clammy. You should:

apply gentle manual traction in line with the limb and reassess for a pulse.

During your assessment of a 29-year-old female with significant deformity to her left elbow, you are unable to palpate a radial pulse. Your transport time to the hospital is approximately 40 minutes. You should

shotgun

During your assessment of a patient who was shot in the abdomen, you notice a large entrance wound with multiple small puncture wounds surrounding it. This wound pattern is MOST consistent with a:

7

During your assessment of a patient with a head injury, you note that he opens his eyes when you pinch his trapezius muscle, is mumbling, and has his arms curled in toward his chest. You should assign him a GCS score of:

suction his oropharynx for up to 15 seconds

During your primary assessment of a 19-year-old unconscious male who experienced severe head trauma, you note that his respirations are rapid, irregular, and shallow. He has bloody secretions draining from his mouth and nose. You should:

instruct him to assist her ventilations while you perform a rapid assessment.

During your primary assessment of a semiconscious 30-year-old female with closed head trauma, you note that she has slow, irregular breathing and a slow, bounding pulse. As your partner maintains manual in-line stabilization of her head, you should:

. stabilize the pelvis with a pelvic binder and protect the spine.

During your secondary assessment of a 30-year-old male who fell 25 feet, you note crepitus when palpating his pelvis. Your partner advises you that the patient's blood pressure is 80/50 mm Hg and his heart rate is 120 beats/min and weak. After completing your assessment, you should:

the speed of the fall

Factors that should be considered when assessing a patient who has fallen include all of the following, EXCEPT: the speed of the fall. the surface struck. the height of the fall. the primary impact point

the patient is clinically unstable

In general, musculoskeletal injuries should be splinted before moving the patient unless:

a complete circuit must exist between the electrical source and the ground.

In order for electricity to flow through the body and cause damage:

pneumothorax

In order to avoid exacerbating a patient's injury, it is especially important to use extreme caution when providing positive-pressure ventilation to patients with a:

A pulseless and apneic patient with a knife impaled in the back

In which of the following patients should you remove an impaled object?

If resistance is encountered or the patient experiences severe pain

In which of the following situations should the EMT splint an injured limb in the position of deformity?

sponatenous pneumothorax

May be the result of rupture of a weak spot on the lung, allowing air to enter the pleural space and accumalate. this often results from nontraumatic injuries and may occur during times of physical activity such as excerise

Best Motor Response

Obeys commands- 6 Localizes pain - 5 Withdraws to pain - 4 Abnormal flexion - 3 Abnormal extension - 2 None - 1

it causes a problem managing the airway

Once a cervical collar has been applied to a patient with a possible spinal injury, it should not be removed unless:

collapsed jugular veins

Signs and symptoms of a tension pneumothorax include all of the following, EXCEPT:

tendons

Skeletal muscle is attached to the bone by tough, ropelike, fibrous structures called:

brain and spinal cord

The central nervous system (CNS) is composed of the:

7

The cervical spine is composed of _____ vertebrae.

transport to a trauma center

The driver of a sport utility vehicle lost control and struck a utility pole head-on. The driver was killed instantly. The passenger, a young female, is conscious and alert and has several small abrasions and lacerations to her left forearm. Treatment for the passenger should include:

kinetic energy

The energy of a moving object is called:

cervical,thoracic,lumbar,sacral, and coccygeal

The five sections of the spinal column, in descending order, are the:

color

The germinal layer of the epidermis contains pigment granules that are responsible for skin:

four-person log roll

The ideal procedure for moving an injured patient from the ground to a backboard is:

your awareness and concern for potentially serious underlying injuries

The index of suspicion is MOST accurately defined as:

bones and voluntary muscles of the body

The musculoskeletal system refers to the

central NS and Peripheral NS

The nervous system of the human body is divided into two anatomic parts

a network of nerve fibers, like cables, that transmit information to and from the body's organs to and from the brain

The peripheral NS includes

waterproofs the skin and keeps it supple

The sebaceous glands produce sebum, a material tha

diaphragm

The thoracic cavity is separated from the abdominal cavity by the:

falls and motor vehicle collisions

Two of the MOST common mechanisms of injury for blunt trauma are:

area of soft-tissue damage surrounding the injury.

With regard to musculoskeletal injuries, the zone of injury is defined as the:

A cervical collar is used in addition to, not instead of, manual immobilization.

Which of the following statements regarding cervical collars is correct? Once a cervical collar is applied, you can cease manual head stabilization. A cervical collar is used in addition to, not instead of, manual immobilization. Cervical collars are contraindicated in patients with numbness to the extremities. The patient's head should be forced into a neutral position to apply a cervical collar

A. Compartment syndrome typically develops within 6 to 12 hours after an injury

Which of the following statements regarding compartment syndrome is correct? A. Compartment syndrome typically develops within 6 to 12 hours after an injury B. In most cases, compartment syndrome develops within a few minutes after an injury. C. Compartment syndrome occurs because of increased pressure within the bone cavity. D. Most cases of compartment syndrome occur following a severe fracture of the pelvis

A. Compromised arterial blood flow leads to crush syndrome and can occur when an area of the body is trapped for longer than 4 hours.

Which of the following statements regarding crush syndrome is correct? A. Compromised arterial blood flow leads to crush syndrome and can occur when an area of the body is trapped for longer than 4 hours. B. Tissue damage that occurs in crush syndrome is severe, but kidney injury is unlikely because toxins are quickly eliminated from the body. C. Provided that a patient with a crush injury is freed from entrapment within 6 hours, the amount of tissue damaged is generally minimal. D. With crush syndrome, massive blood vessel damage occurs following severe soft-tissue injuries, such as amputation of an extremity.

External bleeding may be minimal but internal injuries can be extensive.

Which of the following statements regarding penetrating injuries is correct? External bleeding may be minimal but internal injuries can be extensive. The degree of internal injury can often be estimated by the external injury. It is important to distinguish between entrance and exit wounds in the field. The depth of a penetrating injury should be thoroughly assessed by the EMT.

It is often discovered when the patient is jarred or moved suddenly.

Which of the following statements regarding rebound tenderness is correct? It should be assessed for by vigorously palpating the abdomen. Rebound tenderness is a specific sign found with a spleen injury. It is often discovered when the patient is jarred or moved suddenly. The absence of rebound tenderness rules out intra-abdominal injury.

Severe burns are typically a combination of various degrees of burns

Which of the following statements regarding severe burns is correct? Severe burns involving the airway have a 100% mortality rate. The majority of severe burns involve full-thickness burns only. Patients with severe burns are especially prone to hyperthermia. Severe burns are typically a combination of various degrees of burns

Eighty percent of the cranium is occupied by brain tissue.

Which of the following statements regarding the cranium is correct? The skull is a subdivision of the cranium. Thirty percent of the cranium is occupied by blood. The cranium protects the structures of the face. Eighty percent of the cranium is occupied by brain tissue.

it is the most dramatic part of the collision and may make extrication difficult

Which of the following statements regarding the first collision that occurs during a motor vehicle crash is correct?

Uterine trauma is likely to cause shock in the pregnant patient.

Which of the following statements regarding trauma during pregnancy is correct? Uterine trauma in the pregnant patient rarely leads to severe bleeding. Uterine trauma is likely to cause shock in the pregnant patient. Uterine blood supply is greatly reduced during pregnancy. The uterus serves as a reservoir for blood supply during pregnancy.

rotational and rollover

Which types of motor vehicle collisions present the greatest potential for multiple impacts?

lift the airbag and look for deformity to the steering wheel

While assessing a 21-year-old female who struck a tree head-on with her small passenger car, you note that her air bag deployed. You should:

bruising of the heart muscle

While assessing a young male who was struck in the chest with a steel pipe, you note that his pulse is rapid and irregular. You should be MOST suspicious for:

neck and facial injuries

While en route to a major motor vehicle crash, an on-scene police officer advises you that a 6-year-old male who was riding in the front seat is involved. He further states that the child was only wearing a lap belt and that the air bag deployed. On the basis of this information, you should be MOST suspicious that the child has experienced:

administer oxygen and transport to the hospita

While jogging, a 19-year-old male experienced an acute onset of shortness of breath and pleuritic chest pain. He is conscious and alert with stable vital signs. Your assessment reveals that he has diminished breath sounds over the left side of the chest. You should:

immediately request ALS support.

You respond to a residence for a 40-year-old female who was assaulted by her husband; the scene has been secured by law enforcement. Upon your arrival, you find the patient lying supine on the floor in the kitchen. She is semiconscious with severely labored breathing. Further assessment reveals a large bruise to the left anterior chest, jugular venous distention, and unilaterally absent breath sounds. As your partner is supporting her ventilations, you should:

9

Your patient has a Glasgow Coma Scale (GCS) score of 13, a systolic blood pressure of 80 mm Hg, and a respiratory rate of 8 breaths/min. His Revised Trauma Score (RTS) is:

proxmial femur

a "Hip" fracture is actually a fracture of the

internal bleeding

abdominal rigidity and periumbilical bruising are signs of

pneumothorax Occusive dressing

any penetrating injury to the chest may result in air entering the pleural space and may cause what type of dressing should be used

lateral collisons

approx 25% of severe injuries to the aorta occur during

older people several hours, days or weeks

are at higher risk for a subdural hematoma developing. signs and symptons of the conditon may not occur for

open injuries

are the result of an object penetrating the skin and/or chest wall

transport time, transport destination, and selection of type of transport

caring for victims of traumatic injuries require a solid understanding of the trauma system in the US this includes

rapid, thready pulse

common signs and symptoms of a serious head injury include all of the following, EXCEPT A. widening pulse pressure. B. decerebrate posturing. C. a rapid, thready pulse. D. CSF leakage from the ears.

dressings and bandages

control bleeding, protect the wound from further damage, prevent further contaimination, and prevent infection

minute volume will decrease

if a person's tidal volume decreases, but his or her respiratory rate remains unchanged

hemothorax

is the result of blood accumulating in the plueral space after a traumatic injury when the vessels of the lung are lacerated and leak blood

simple pneumothorax

is the result of blunt trauma, such as fractured ribs

may be an indicator of deeper, more serious injuries

lacerations to the scalp

commotio cordis

occurs from direct blows to the chest during a critical portion of the patient's heartbeat. it may result in immediate cardiac arrest

accumulation of air in the pleural space

pneumothorax is defined as:

strain

stretching of the muscle

ventilation

the body's ability to move air in and out of the lungs is called

autonomic

the body's functions that occur without conscious effort are regulated by the ___ nervous system

Brain and spinal cord

the central NS is composed of

Meninges bony structures

the central NS is protected by layers of tissue called and also protected by

mediastinum

the esophagus, trachea, and great vessels of the body are located in the

blunt or penetrating trauma

traumatic injuries can be described as


Kaugnay na mga set ng pag-aaral

Human Development Chapter's 13-16

View Set

Health Assessment Ch 16: assessing the eyes

View Set

Lesson 5 Quiz: Real Estate Leases

View Set

Chapter 5: health insurance policy provisions

View Set